The Psychiatric-Mental Health Nurse Practitioner Certification Review Manual Flashcards
2-01. Which of the following is not a core competency of APRNs?
A. Technology and information literacy
B. Health delivery system
C. Ethics
D. Delegated practice
Answer D. Delegated practice is not a core competency of the APRN. The nurse practitioner may delegate individual tasks but not the practice. The APRN must demonstrate competency in technology and information literacy, health delivery systems, and ethical principles and decision making.
2-02. Which of the following is characteristic of the leadership competency for APRNs?
A. Participating in a community-focused program that promotes mental health and reduces the risk of mental health problems
B. Evaluating the appropriate use of seclusion and restraints in caring for a mentally ill patient
C. Developing an age-appropriate treatment plan
D. Modifying the treatment plan based on the patient’s needs
Answer A. Participating in a community-focused program that promotes mental health exemplifies a leadership competency. Evaluating the appropriate use of seclusion and restraints, developing an age-appropriate treatment plan, and modifying the treatment plan based on clients’ needs are clinical competencies
2-03. All of the following are characteristic of the independent practice competency except:
A. Insulin during differential diagnoses
B. Conducting individual and group psychotherapy
C. Taking opportunities to influence health policy to reduce the stigma of mental health services
D. Implementing plans to minimize complications and to promote function
Answer C. Taking opportunities to influence health policy and reduce the stigma of mental health services exemplifies health policy competency. Considering differential diagnoses, conducting psychotherapy, and implementing plans to minimize comorbidity are examples of practicing with full practice authority at the top of the professional license.
2-04. A state’s Nurse Practice Act specifies all of the following except:
A. Who may use the title nurse practitioner
B. What a nurse practitioner can do
C. Restriction of the nurse practitioner’s practice
D. Provide oversight for service and benefits provided to patients
Answer D. The state Nurse Practice Act does not provide for oversight of services and benefits provided to patients. It does define who may use the title and the work of the nurse practitioner as well as restrict the practice of the professional.
2-05. Which of the following may govern the profession of the PMHNP?
A. Statutory law
B. The U.S. Constitution
C. Certification
D. City ordinances
Answer A. Statutory laws govern the practice of a profession. The U.S. Constitution, certifications, and city ordinances do not address professional practice.
2-06. Which of the following factors are facilitating the growth of the PMHNP?
A. Increased competition in the job market
B. Overlapping scope of practice with other types of nurse practitioners
C. Decreasing stigmatization
D. Collaborative practice relationships with physicians
Answer C. Decreasing stigmatization regarding mental health enhances the growth of the profession. Increased competition, overlapping scope of practice, and mandated collaborative practice relationships restrict the growth of the profession.
2-07. Why is it necessary for insurance companies to credential PMHNPs?
A. To protect the public by ensuring a minimum level of professional competence
B. To define the scope of practice and practice requirements
C. To ensure a professional organization certifies the person is licensed to practice
D. To define the role of the nurse practitioner
Answer A. The purpose of credentialing is to ensure public safety and a minimum level of professional competence; this helps reduce vicarious liability on the insurance company. Insurance 35companies do not define the scope of practice; they have nothing to do with professional organizations who grant certification. State governing bodies define the role of the nurse practitioner.
2-08. Which of the following allows the PMHNP to practice in a state?
A. Certification
B. Licensure
C. Credentialing
D. Graduation
Answer B. Certification allows a professional to practice their profession in a state. Certification designates successful completion of a course of study. Credentialing is a process that verifies minimum levels of professional competence to ensure public safety. Graduation is a ceremony offered by an educational institution in accordance with established criteria of completion.
2-09. All of the following statements are true regarding scope of practice except:
A. The scope of practice defines the nurse practitioner role and actions
B. The scope of practice identifies competencies assumed to be held by all nurse practitioners in a specific role
C. The scope of practice provides a way to judge the nature of the care provided
D. The scope of practice varies broadly from state to state
Answer C. The scope of practice defines the nurse practitioner role and actions, identifies minimal competencies held by all nurse practitioners, and varies broadly from state to state. It does not provide a way to judge the nature of care provided.
2-10. All the following statements are true regarding standards of practice except:
A. Standards of practice reflect professional agreement focusing on the minimum levels of acceptable performance
B. Standards of practice provide a way to judge the nature of care provided
C. Standards of practice identify competencies assumed to be held by all nurse practitioners (NPs) who function in a similar role
D. Standards of practice can be used legally to describe a standard of care that must be met by a provider
Answer C. Standards of practice reflect the professional agreement that focuses on the minimum levels of acceptable performance, a means of judging the nature of care provided, and can be used legally to describe the standard of care that must be met by a provider. It does not identify competencies held by all nurse practitioners as that is provided in the scope of practice.
2-11. All of the following are considered personal health identifiers except:
A. Patient’s name
B. Patient’s diagnosis
C. Address
D. Phone number
Answer B. Personal health identifiers include name, address, and phone number. Patient diagnosis is not considered a personal health identifier.
2-12. Unless the patient provides consent for releasing information, the Health Information Portability and Accountability Act (HIPAA) prohibits the disclosure of personal health identifiers to all of the following except:
A. The patient’s next of kin
B. The patient’s health insurance company
C. The patient’s clergy
D. The patient’s attorney
Answer B. The patient must consent to release personal health identifiers to next of kin, clergy, and an attorney. Patient consent is not required to release personal health identifiers to a patient’s health insurance company.
2-13. All of the following circumstances would allow the release of medical information without the patient’s consent except:
A. If the client reveals intent to harm self or others
B. Records to the insurance company
C. Responding to court orders, subpoenas, or summons
D. Next of kin wanting to help the patient make decisions
Answer D. Medical information may be released to parties needing to know to prevent harm to self and others, arranging for payment from the health insurance company, and in response to court subpoenas or summonses. The patient must consent to release medical information to the next of kin helping to make healthcare decisions.
2-14. A PMHNP is asked to determine if the patient has the capacity for informed consent. All of the following elements are necessary for making this determination except:
A. The patient understands the purpose of the proposed treatment for the procedure
B. The patient understands the risks and discomforts and benefits of the treatment
C. The patient understands alternatives to the intended procedure
D. The patient has a therapeutic alliance with the surgeon
Answer D. The following elements must be considered when asked to determine capacity for informed consent: the patient’s understanding of the purpose of the proposed treatment, the risks and discomforts, benefits of the treatment, and alternatives to the intended procedure. The therapeutic alliance with the surgeon is not essential for a capacity decision.
2-15. Which of the following ethical principles deals with promoting the well-being of a patient?
A. Justice
B. Beneficence
C. Nonmaleficence
D. Autonomy
Answer B. The principle of beneficence is that of doing good for the patient. Justice is the principle of equity. Nonmaleficence is the principle of first doing no harm. Autonomy is the principle of respecting the individual’s right to self-determination.
2-16. Which of the following ethical principles would conflict if the treatment team wanted to implement a therapeutic deception for the good of the patient?
A. Beneficence
B. Nonmaleficence
C. Justice
D. Veracity
Answer D. Veracity is the ethical principle of truthfulness and nondeception neither by omission or commission. Beneficence is the principle of doing good. Nonmaleficence is the principle of first doing no harm. Justice is the principle of equity.
2-17. A patient who is refusing psychiatric treatment has their case adjudicated. The judge orders treatment over objection for the good of the patient. Which ethical principle is trumped?
A. Beneficence
B. Nonmaleficence
C. Justice
D. Autonomy
Answer D. Autonomy is the principle of respecting the individual’s right to self-determination. Beneficence is the principle of doing good. Nonmaleficence is the principle of first doing no harm. Justice is the principle of equity.
2-18. Actions judged based on their inherent value regardless of their consequence is based on which ethical decision-making approach?
A. Teleological theory
B. Deontological theory
C. Virtue ethics
D. Justice principle
Answer B. Deontological theory considers the inherent value regardless of consequences. The teleological approach allows the ends to justify the means. Virtue ethics is an approach to ethical decision-making that considers the moral character of the individual rather than the action. Justice is the principle of equity.
2-19. An action that is judged good or bad based on the consequence demonstrates which ethical decision-making approach?
A. Teleological theory
B. Deontological theory
C. Virtue ethics
D. Justice principle
Answer A. The teleological approach allows the ends to justify the means. Deontological theory considers the inherent value regardless of consequences. Virtue ethics is an approach to ethical decision-making that considers the moral character of the individual rather than the action. Justice is the principle of equity.
2-20. All of the following are necessary criteria for commitment for involuntary evaluation and treatment except:
A. The person has a diagnosed psychiatric disorder
B. The person has potential to harm self or others because of the disorder and is unaware or unwilling to accept the nature and severity of the disorder
C. Treatment is likely to improve function
D. The patient is unable to afford the cost of treatment
Answer D. 36Involuntary commitment is intended to protect the patient and society; therefore, the patient’s financial resources are not considered as a criteria for treatment.
2-21. A PMHNP is working with a nonprofit organization to implement a public health campaign aimed at reducing the stigma of mental illness. Which role of the PMHNP best describes this activity?
A. Mentoring
B. Advocacy
C. Policy making
D. Case management
Answer B. Advocacy entails promoting a particular cause or policy. Mentoring is a relationship with a more experienced individual for the purpose of guiding the professional development of the less experienced individual. Policy making is the process of developing laws and regulations. Case management is a collaborative process of assessment, planning, facilitation, care coordination, and evaluation of resources needed to meet the patient’s needs in a cost-effective manner.
2-22. The PMHNP is working with an insurance organization to provide oversight and authorization of series and benefits to patients. Which role of the PMHNP best describes this activity?
A. Mentoring
B. Advocacy
C. Policy making
D. Case management
Answer D. Case management is a collaborative process of assessment, planning, facilitation, care coordination, and evaluation of resources needed to meet the patient’s needs in a cost-effective manner. Mentoring is a relationship with a more experienced individual for the purpose of guiding the professional development of the less experienced individual. Advocacy entails promoting a particular cause or policy. Policy making is the process of developing laws and regulations.
2-23. Which of the following roles of the PMHNP requires mutual respect and an interactive learning process to facilitate role development, competencies, and skill acquisition?
A. Mentoring
B. Advocacy
C. Policy making
D. Case management
Answer A. Mentoring is a relationship with a more experienced individual for the purpose of guiding the professional development of the less experienced individual. Advocacy entails promoting a particular cause or policy. Policy making is the process of developing laws and regulations. Case management is a collaborative process of assessment, planning, facilitation, care coordination, and evaluation of resources needed to meet the patient’s needs in a cost-effective manner.
2-24. All of the following are true regarding living wills except:
A. Legally binding in all 50 states
B. Designates in writing an agent to act on behalf of the person should they become unable to make decisions
C. Prepared while the client is mentally competent to designate preferences for care if the client becomes incompetent
D. Should be considered as an aspect of relapse planning for compliance with chronic psychiatric disorders
Answer A. A living will is not legally binding in all 50 states, but does designate agents to act on behalf of the patient in the event of incapacity, is prepared while the patient has capacity, and is considered an aspect of relapse planning for compliance with chronic psychiatric disorders.
2-25. A specific behavior related to a person’s culture and not leading to a psychiatric disorder is classified as:
A. Temporary delusion
B. Brief psychotic episode
C. Adjustment disorder
D. Culture-bound syndrome
Answer D. Culture-bound syndrome is specific behavior common to a person’s culture, and is considered a variation of normal and not a psychiatric diagnosis. Delusions are deeply held beliefs despite evidence to the contrary. A brief psychotic episode is an acute onset thought disorder marked by disorganization and at least one of the following symptoms: delusions, hallucinations, bizarre behavior and posture, and disorganized speech. Adjustment disorder is a mood disorder marked by emotional or behavioral symptoms that develop in response to an identifiable stressor within 3 months of onset of the stress and lasting no more than 6 months.
2-26. The learned beliefs and behaviors common among all members of a group defines:
A. Complex delusions
B. Folie a deux
C. Culture
D. Folie a famille
Answer C. Culture is the learned belief and behaviors common among all members of a group. Complex delusions are deeply held beliefs despite evidence to the contrary with an elaborate web of interconnected delusions. Folie a deux is a shared delusion between two people. Folie a famille is a delusion held among cohabitating family members.
2-27. All of the following factors put households headed by a single female at risk for homelessness except:
A. Education
B. Underemployment
C. Inadequate benefits
D. Inaccessible housing options
Answer A. Education increases the likelihood of gainful employment. Underemployment, inadequate benefits, and inaccessible housing all increase the risk of homelessness.
2-28. Which mental illness accounts for the highest rate of mental illness?
A. Major depressive disorder
B. Bipolar disorder
C. Schizophrenia
D. Oppositional defiant disorder
Answer A. The incidence of major depressive disorder in the United States is 12.8%, oppositional defiant disorder is 10.2%, bipolar disorder 2.6%, and schizophrenia 1%.
2-29. During a psychiatric evaluation, the client reveals to the PMHNP a transgender identity. The client indicates a preference for personal pronouns like he and him. What is the biological sex of the client?
A. Male
B. Female
C. Lesbian
D. Asexual
Answer B. The biological sex of the client is female, but the gender identity is male. Lesbian and asexual are descriptors of sexual orientation.
2-30. A new graduate PMHNP is considering a job at a psychiatric ED. The medical director informs the prospective hire that they are expected to evaluate and treat children, adolescents, adults, and geriatric patients. What document should be consulted to determine if the PMHNP can legally treat psychiatric patients across the life span?
A. The state Nurse Practice Act
B. The hospital bylaws
C. The PMHNP scope of practice
D. The PMHNP can treat any patient because there is a collaborating physician
Answer A. A state Nurse Practice Act describes what is permissible and impermissible for a PMHNP to do. The hospital bylaws govern the local organization and can be more restrictive, but not more lenient than state laws. The scope of practice identifies competencies unique to the subspecialty. The PMHNP is an independently licensed individual who bears liability for errors of omission and commission regardless of the collaborative agreement. The collaborative agreement cannot supersede state law.
2-31. The process of synthesizing, disseminating, and using research-generated knowledge to make a change in practice is known as:
A. Research utilization
B. Evidence-based practice
C. Research
D. Quality improvement
Answer A. Research utilization is the process of synthesizing and disseminating research-generated knowledge to make a change in care. Evidence-based practice is a problem-solving approach that uses the best evidence in making decisions about patient care that takes into account client needs, values, and clinical expertise, but may not be generalizable beyond the immediate problem. Research is the systematic empirical inquiry of relationships among variables. Quality improvement is a systematic approach to the analysis of current practice, efficiency, outcomes, and benchmarks.
2-32. The process of integrating the best research evidence with clinical expertise and client needs and values is known as:
A. Research utilization
B. Evidence-based practice
C. Research
D. Quality improvement
Answer B. Evidence-based practice is a problem-solving approach that uses the best evidence in making decisions about patient care that takes into account client needs, values, and clinical expertise, but may not be generalizable beyond the immediate problem. Research utilization is the process of synthesizing and disseminating research-generated knowledge to make a change in care. Research is the systematic empirical inquiry of relationships among variables. Quality improvement is a systematic approach to the analysis of current practice, efficiency, outcomes, and benchmarks.
2-33. All of the following are components of research utilization except:
A. Critique of the literature
B. Synthesize findings
C. Applying the findings
D. Randomizing the sample
Answer D. Randomizing the sample is a component of certain types of research. Critique and literature synthesis and applying the findings are components of research utilization.
2-34. The first step in implementing an evidence-based practice change is to:
A. Identify the patient, population, and problem
B. Identify key shareholders
C. Obtain institutional review board approval
D. Review the literature
Answer A. Identifying the patient, population, and problem is the first step in implementing an evidence-based practice change. Identifying stakeholders, obtaining institutional review board approval, and reviewing the literature are subsequent steps.
2-35. All of the following are methods of dissemination of findings except:
A. Presenting at conferences
B. Publishing in peer-reviewed journals
C. Poster presentation
D. Oral defense to your committee
Answer D. Oral defense to your committee is not a method of dissemination, but rather peer review. Presenting at conferences, publishing, and poster presentations are examples of dissemination.
2-36. What concept is defined by a sample that is representative of the population, and where the results can be generalized?
A. Internal validity
B. External validity
C. Prevalence
D. Incidence
Answer B. External validity relates to the generalizing of findings beyond the study to the population. Internal validity is the degree of truthfulness about inferences regarding causal relationships. Prevalence is a measure of all individuals affected at a particular time. Incidence is a measure of the number of new occurrences only during a particular period of time.
2-37. What concept is defined by an independent variable causing a change in the dependent variable?
A. Internal validity
B. External validity
C. Prevalence
D. Incidence
Answer A. Internal validity is the degree of truthfulness about inferences regarding causal relationships. External validity relates to the generalizing of findings beyond the study to the population. Prevalence is a measure of all individuals affected at a particular time. Incidence is a measure of the number of new occurrences only during a particular period of time.
2-38. What method of analysis describes the basic features of the data and summarizes and organizes observations?
A. Inferential statistics
B. Descriptive statistics
C. Regression analysis
D. Analysis of variance (ANOVA)
Answer B. Descriptive statistics define, summarize, and organize data. Inferential statistics use various methods of analysis to deduce properties of probability and distribution that may be reflective of the population. Regression analysis is a statistical method that allows one to examine the relationship between two or more variables. ANOVA is a collection of statistical models and their associated estimation to analyze differences among group means in a sample.
2-39. What method of analysis enables one to reach conclusions that extend beyond the immediate data and is generated by quantitative research designs?
A. Inferential statistics
B. Descriptive statistics
C. Regression analysis
D. Standard deviations
Answer A. Inferential statistics use various methods of analysis to deduce properties of probability and distribution that may be reflective of the population. Descriptive statistics define, summarize, and organize data. Regression analysis is a statistical method that allows one to examine the relationship between two or more variables. Standard deviation is a calculated quantity to indicate the degree of spread among data.
2-40. Which statistical test differentiates among three or more groups?
A. t-test
B. Analysis of variance (ANOVA)
C. Pearson’s r correlation
D. Probability
Answer B. ANOVA is a collection of statistical models and their associated estimation to analyze differences among group means in a sample. A t-test is a type of inferential statistic used to determine if there is a significant difference in means between two groups assuming normal distribution. Pearson’s r correlation is a number between −1 and 1 that indicates the extent to which two variables are linearly related. Probability is a ratio describing the extent to which something is likely to occur.
2-41. Which statistical test identifies if the means of two groups are statistically different from each other?
A. t-test
B. Analysis of variance (ANOVA)
C. Pearson’s r correlation
D. Probability
Answer A. A t-test is a type of inferential statistic used to determine if there is a significant difference in means between two groups assuming normal distribution. ANOVA is a collection of statistical models and their associated estimation to analyze differences among group means in a sample. Pearson’s r correlation is a number between −1 and 1 that indicates the extent to which two variables are linearly related. Probabilityis a ratio describing the extent to which something is likely to occur.
2-42. Which statistical test identifies a relationship between two variables?
A. t-test
B. Analysis of variance (ANOVA)
C. Pearson’s r correlation
D. Probability
Answer C. Pearson’s r correlation is a number between −1 and 1 that indicates the extent to which two variables are linearly related. A t-test is a type of inferential statistic used to determine if there is a significant difference in means between two groups assuming normal distribution. ANOVA is a collection of statistical models and their associated estimation to analyze differences among group means in a sample. Probability is a ratio describing the extent to which something is likely to occur.
2-43. The level of significance describing the probability of a particular result occurring by chance alone is represented by which of the following?
A. p-value
B. z-score
C. Odds ratio
D. Confidence interval
Answer A. The p value describes the level of significance describing the probability of a particular result occurring by change alone. The z-score is a standardized measure of standard deviations above or below a population mean. The odds ratio is a statistic that attempts to quantify the strength of an association between an exposure and an outcome. The confidence interval describes a range of values indicating the likelihood of the data containing a true value of an unknown population.
2-44. All of the following are governed by the institutional review board except:
A. Ensuring risks to participants are minimized
B. Ensuring equitable selection
C. Evaluating risks and benefits, ensuring reporting of adverse events
D. Regulating funding for the study
Answer D. The institutional review board does not govern funding of a study, but does ensure risks to participants are minimized, subjects are equitably selected, and ensures the reporting of adverse events and risk-benefits are reevaluated.
2-45. All patients have the right to all of the following except:
A. Confidential care
B. Care in the least restrictive manner
C. The ability to consent and withdraw consent at any time
D. To be treated by a physician
Answer D. Clients have a right to be treated by a qualified provider who may not be a physician. All patients have the right to receive care in the least restrictive manner with the utmost confidence, and may consent or withdraw consent at any time.
3-01. A member of the group states that his problem is not that bad and he can stop anytime he chooses. Using on occasion is just a way to have fun with friends. Which concepts of the Health Belief Model best describes the group member’s readiness to change?
A. Perceived severity and perceived barriers
B. Self-efficacy and perceived benefits
C. Cues to action and perceived susceptibility
D. Self-efficacy and perceived susceptibility
Answer A. The member is expressing the belief regarding the seriousness of the condition as minimal, and that the cost of abstaining from the addiction is reducing time with friends. Self-efficacy is an individual’s belief in his or her innate ability to achieve a goal and perceived benefits refer to the individual’s perception of the effectiveness of an action to reduce a threat or illness. Perceived susceptibility is the individual’s perception of acquiring disease or illness, and cues to action are stimuli necessary to trigger a health-promoting behavior.
3-02. A member of the group expresses how difficult it has been to maintain her sobriety and as a result does not feel she will be successful this time. Which concept of the Health Belief Model would be the most significant risk factor for relapse?
A. Low perception of severity
B. Low self-efficacy
C. High perceived benefits
D. Cues to action
Answer B. The member is expressing the belief that because she has relapsed in the past, that she is likely to relapse again, which reflects a low confidence in her ability to successfully perform the behavior. Perceived benefits refer to the individual’s perception of the effectiveness of an action to reduce a threat or illness. Perceived susceptibility is the individual’s perception of acquiring disease or illness, and cues to action are stimuli necessary to trigger a health-promoting behavior.
3-03. A member attending the group for the first time expresses gratitude for being court ordered to attend the recovery group instead of serving prison time, and he realizes how out of control he was while using. Which Health Belief Model concept was demonstrated?
A. Perceived severity and perceived barriers
B. Self-efficacy and perceived benefits
C. Cues to action and perceived severity
D. Self-efficacy and perceived susceptibility
Answer C. The member describes the trigger that activated the ability to take action, and the seriousness of the condition and sequelae. Perceived barriers are the belief in the tangible and psychological costs associated with the advised behavior. Perceived severity is the individual’s belief of how serious a condition and its consequences are. Self-efficacy is an individual’s belief in his or her innate ability to achieve a goal.
3-04. A 30-year-old man with three past psychiatric hospitalizations for schizoaffective disorder presents for follow-up care with the PMHNP. Before discharge, the patient received court ordered Zyprexa Relprevv 300 mg IM X 1. The patient tells the PMHNP that he is aware that smoking can “make him need more medication.” Which stage of change is the patient in?
A. Precontemplation
B. Contemplation
C. Preparation
D. Action
E. Maintenance
Answer B. He is in the contemplation stage: The patient has indicated that a problem exists but has not expressed a desire to change. Precontemplation—In this stage, people do not intend to take action in the foreseeable future (defined as within the next 6 months). Preparation (determination)—In this stage, people are ready to take action within the next 30 days. Action—In this stage, people have recently changed their behavior (defined as within the last 6 months) and intend to keep moving forward with that behavior change. Maintenance—In this stage, people have sustained their behavior change for a while (defined as more than 6 months) and intend to maintain the behavior change going forward.
3-05. The PMHNP is caring for a 20-year-old man with bipolar I disorder. The patient indicates that he does not have a problem because he can accomplish so much during his manic episodes. Which stage of change is the patient in?
A. Precontemplation
B. Contemplation
C. Preparation
D. Action
E. Maintenance
Answer A. The patient is in the precontemplation stage; in this stage, people do not intend to take action in the foreseeable future (defined as within the next 6 months). The patient denies the problem and has no intention of making a change. Contemplation—In this stage, people intend to start the healthy behavior in the foreseeable future (defined as within the next 6 months). Preparation (determination)—In this stage, people are ready to take action within the next 30 days. Action—In this stage, people have recently changed their behavior (defined as within the last 6 months) and intend to keep moving forward with that behavior change. Maintenance—In this stage, people have sustained their behavior change for a while (defined as more than 6 months) and intend to maintain the behavior change going forward.
3-06. The PMHNP is explaining the risks and benefits of long-acting injectable antipsychotics to a patient with a history of medication nonadherence. The patient is asking logical questions that demonstrate he can understand the facts. Which process of changing represents the patient’s position?
A. Social liberation
B. Reinforcement management
C. Self-liberation
D. Consciousness raising
Answer D. The patient is in the consciousness raising stage; i.e., increasing awareness about healthy behavior and discovering and accepting facts in support of the health behavior change. Social liberation—Environmental opportunities that exist to show society is supportive of healthy behavior. Reinforcement management—Rewarding the positive behavior and reducing the rewards that come from negative behavior. Self-liberation—Commitment to change behavior based on the belief that achievement of the healthy behavior is possible. Consciousness raising—Increasing awareness about healthy behavior.
3-07. The PMHNP has prescribed a patient naltrexone and bupropion for binge eating disorder. The patient has lost 4 lbs. since her last visit 1 month ago. The patient states that sometimes she finds it very difficult to not binge eat when her emotions are intense, and expresses doubt in her ability to make a lasting change. Which of the following is the best response?
A. “I can increase your dose of medication or augment you with another medication.”
B. “On a scale of 1 to 10 how important is this change for you to make?”
C. “Give me an example of something in your life that you are most proud of achieving.”
D. “Would you like a referral to a dietician?”
Answer C. The patient is expressing low self-efficacy; by asking about past successes the patient’s self-efficacy may be enhanced. Although augmenting the medication may be possible, the patient is experiencing the positive effects of the medication (weight loss).
3-08. A 35-year-old female with personality disorder not otherwise specified takes quetiapine 50 mg qhs for sleep, and 12.5 mg q6h PRN for anxiety. She tells the PMHNP that no one seems to understand how much stress she is under and would like to change providers. Which of the following would be the best response?
A. “Do you think you need your medication adjusted?”
B. “It sounds to me like you are frustrated and do not feel listened to.”
C. “I can give you a list of three psychiatrists whom you can choose from to give you the care you want.”
D. “It will take some time for the medication to reach full effect before you are feeling better.”
Answer B. Express empathy by affirming the client’s feelings, and demonstrating reflective listening by using her words to gain insight into her motivations. Asking if she needs her medication adjusted invalidates her feelings as does giving her an answer about medication efficacy. Offering her a list of referrals communicates a lack of commitment to the therapeutic relationship on the part of the PMHNP if done without exploring underlying motivation.
3-09. Which of the following is a therapeutic technique to facilitate a health behavior change in a 28-year-old male with substance use disorder who is in denial regarding the severity of his habit?
A. Solution-focused therapy
B. Strategic therapy
C. Psychopharmacology with naltrexone 50 mg daily
D. Motivational interviewing
Answer D. Motivational interviewing will allow the PMHNP to hear the client’s reasons for maintaining the status quo as well as elicit his reasons for what it would take to trigger a desire to change. Solution-focused therapy is a goal-directed psychotherapy based on direct observation to a series of precisely constructed questions. Strategic (family) therapy does not focus on the individual but the social structure. Psychopharmacology may be appropriate but the naltrexone is not indicated as the substance is not specified.
3-10. A 65-year-old female was discovered in her apartment by police, who called an ambulance for transport to the hospital, after her neighbors had not seen her in several days. The patient received IV hydration, and her labs show microcytic hypochromic anemia. She is admitted to inpatient psychiatry for evaluation and treatment of what is thought to be a hoarding disorder. Which need is of primary concern to the PMHNP?
A. Adequate nutrition, hydration, and safety planning
B. The patient feeling a sense of belonging
C. The patient attends psychosocial rehab groups
D. The patient requires forms for the Family Medical Leave Act
Answer A. Physiological needs must take priority over psychological needs. According to Maslow’s hierarchy physiological needs include sex, food, water, warmth, and rest, followed by safety needs, then psychological needs of love and belonging, esteem needs including prestige and feelings of accomplishment, and finally self-actualization defined as achieving one’s full potential and using creative energy.
3-11. A 60-year-old man with alcohol use disorder in remission presents to the psychiatric ED and is very distraught. He tells the PMHNP that he was at his daughter’s wedding and called upon to make a toast. The waiter had handed him the champagne glass, and he took a sip of alcohol for the first time in 10 years. He tells the PMHNP that he is concerned he may relapse and go back to his old ways. What is the best response by the PMHNP?
A. “Don’t worry; it was only one drink.”
B. “How have you been able to maintain your sobriety for so long?”
C. Tell the patient he will be admitted to inpatient rehab.
D. Start the patient on naltrexone 50 mg daily for alcohol use disorder
Answer B. Enhancing the patient’s self-efficacy by having him reflect on the effective strategies that have been previously working for him will reduce his risk of relapse. Telling a patient not to worry is invalidating. It is not appropriate to admit a patient to inpatient rehab as he has not relapsed but lapsed. Starting the patient on naltrexone for daily use may be appropriate as part of enhancing sobriety, but the Sinclair method, in this case, is a more tailored response and does not negate his adaptive coping mechanisms for the last 10 years.
3-12. A 25-year-old female, admitted to an extended observation psychiatric unit, observes that another patient is placed on 1:1 observation status after expressing suicidal ideations to her psychiatrist. During her meeting with the PMHNP, the patient tells him that she would like to have 1:1 observation because she too is feeling suicidal. What is the best response by the PMHNP?
A. “What is your plan to commit suicide?”
B. “I will place you on 1:1 status, your safety is very important to us.”
C. Order Haldol 5 mg, and Ativan 2 mg PO X 1 now.
D. Put the patient in therapeutic seclusion to prevent self-harm.
Answer A. Patients may express suicidal ideation, but the PMHNP must assess further to determine lethality, means, coping strategies, and protective factors. Before prescribing a treatment, a complete assessment is necessary.
3-13. A middle-school girl who is timid to take any risks, prefers solitary activities, and does not ask questions of her teachers for fear that she is bothering them likely did not master which developmental stage?
A. Trust versus mistrust
B. Autonomy versus shame and doubt
C. Industry versus inferiority
D. Initiative versus guilt
Answer D. Failing to master the stage of initiative versus guilt inhibits the development of purpose and virtue, and children remain followers, lack self-initiative, and can feel like they are a bother to others. Middle-school children are in the stage of development of industry versus inferiority and moving toward identity versus role confusion.
3-14. A 66-year-old registered nurse, who retired from psychiatry 3 years ago, spends much of her time watching television. She is easily irritated by those interrupting her shows and has no desire to engage in any activities or identify with a broader community. Which developmental stage did she fail to master?
A. Generativity versus stagnation
B. Ego integrity versus despair
C. Industry versus inferiority
D. Initiative versus guilt
Answer A. Mastering the developmental stage of generativity versus stagnation leads to the development of caring as a virtue and a desire to be a part of a broader community, leave a legacy, and feel useful. Failing to master this stage yields a lack of interest in the broader community, and feelings of leading a disconnected existence with a nothing to contribute.
3-15. A woman who discovers her husband has committed suicide in the middle of the night decides to tell her 4- and 6-year-old children the next morning that the reason daddy is not here today is that he went to work. Based on their developmental stage, what might they assume when mommy says she is going to work?
A. Going to work means you are not coming home
B. Mommy will come home, as has been the case in the past
C. When someone cannot be seen, then they cease to exist
D. Daddy will come home when mommy goes to work
Answer A. Children in this stage are considered preoperational and have an organized schema that has helped them understand that when people go to work, they come back. Thinking at this stage is egocentric and magical. Introducing this new accommodation to the children will make them think that when mommy goes to work, she will not be returning home.
3-16. A mother of a teenage boy is unable to acknowledge that her son’s mental illness is perpetuated by his poly substance use disorder and is insisting that his father’s genes are to blame. Which defense mechanism is employed?
A. Reaction formation
B. Regression
C. Denial
D. Dissociation
Answer C. Denial is the not acknowledging an ego dystonic reality. Reaction formation is displacing angry feelings with extremely warm feelings. Regression is characterized by the individual abandoning age-appropriate coping strategies in favor of an earlier, more childlike pattern (e.g., high-pitched voice with up speaking). Dissociation is detaching from the immediate stress, in reality, to protect the ego from a complete loss of reality as in psychosis.
3-17. A 36-year-old Korean male who has recently started attending graduate school complains of persistent abdominal pain despite an extensive gastrointestinal work-up including an upper and lower gastroscopy. Which of the following defense mechanisms is employed?
A. Conversion
B. Dissociation
C. Denial
D. Reaction formation
Answer A. Conversion disorder is the manifestation of physical symptoms due to psychological stressors or angst. Dissociation is detaching from the immediate stress, in reality, to protect the ego from a complete loss of reality as in psychosis. Denial is not acknowledging an ego dystonic reality. Reaction formation is displacing angry feelings with extremely warm feelings.
3-18. The PMHNP is treating a client for generalized anxiety disorder with depressive symptoms. Treatment has included both psychopharmacology and interpersonal psychotherapy. The patient abruptly reports a reemergence of the symptoms that have been in remission. In what phase of therapy is this an expected finding?
A. Orientation
B. Working/treatment
C. Termination
D. Acclimation
Answer C. During the termination phase of a therapeutic relationship, the client may experience reemergence of symptoms previously in remission as a manifestation of regression and resistance to termination. During the orientation phase, the patient may demonstrate symptoms of a reluctance to disclose, avoidance, guardedness, ambivalence, and elation. During the working treatment phase the patient may manifest transference, increased resistance, increased ambivalence, and threaten early termination.
3-19. All of the following are attributes of cultural care that may influence the therapeutic alliance except
A. Religion
B. Gender roles
C. Beliefs
D. Urban setting
Answer D. The aspects of cultural care that may influence the therapeutic alliance include the patient’s values, beliefs, generational influences, gender roles, and religion. The current physical location of the episode of care is not an influencing cultural factor.
3-20. Anxiety can arise from intrapsychic conflict among the structures of the mind. Which structure of the mind deals with reality testing and the thinking process?
A. Id
B. Ego
C. Superego
D. Defense mechanisms
Answer B. The Ego is the structure of the conscious, rational mind that deals with reality testing and thinking processes. The Id is the primitive, pleasure-driving structure, and the Superego is the mediator between the Id and the Ego and deals with morality.
4-01. The PMHNP is evaluating a 68-year-old man for symptoms of major depression. While assessing the appearance of the patient, an obvious left-sided ptosis is noted. Which cranial nerve is directly linked to this condition?
A. III
B. IV
C. V
D. VI
Answer A. Cranial nerve III oculomotor coordinates eye movements including eye lid function and pupillary dilation. Cranial IV (throchlea) allows for the downward medial gaze, V (trigeminal nerve) is associated with the mandible function of mastication, VI (abducens) allows for eye movement in the downward motion.
4-02. A 55-year-old woman has been referred for a psychiatric evaluation for a change in personality, lack of impulse control, and increasingly reckless decision-making. Which area of the brain would be responsible for these symptoms?
A. Parietal lobe
B. Broca’s area
C. Wernicke’s area
D. Frontal lobe
Answer D. Frontal lobe controls voluntary movement, ability to project future consequences, governs social cues, reasoning, planning, and parts of speech. The parietal lobe is associated with sensory integration, attentiveness, and spatial awareness. Broca’s area (although located in the frontal lobe of the dominant hemisphere) specifically is associated with speech production (fluency); Wernicke’s area is associated with comprehension.
4-03. A patient was recently started on a dopamine antagonist, and complains of breast tenderness and a milky discharge. What is the most likely explanation for these symptoms?
A. Dopamine transmission in the nigrostriatal pathway
B. Dopamine transmission in the tuberoinfundibular pathway
C. Dopamine transmission in the mesolimbic pathway
D. Dopamine increases throughout the basal ganglia
Answer B. The tuberoinfundibular pathway transmits dopamine from the hypothalamus to the pituitary gland and increases the secretion of prolactin releasing hormone, in turn creating hyperprolactinemia. The nigrostriatal pathway transmits dopamine from to the midbrain to the caudate nucleus which can trigger various movement disorders. The mesocorticllimbic pathway transmits dopamine to the prefrontal cortex and midbrain, and disorders in this pathway are associated with thought disorders.
4-04. A patient who was recently started on haloperidol for psychosis begins to manifest hand tremor, and a shuffling gait. What is the most likely explanation for these symptoms?
A. Dopamine transmission in the nigrostriatal pathway
B. Dopamine transmission in the tuberoinfundibular pathway
C. Dopamine transmission in the mesolimbic pathway
D. Dopamine increases throughout the basal ganglia
Answer A. The nigrostriatal pathway transmits dopamine from to the midbrain to the caudate nucleus which can trigger various movement disorders. The tuberoinfundibular pathway transmits dopamine from the hypothalamus to the pituitary gland and increases the secretion of prolactin releasing hormone, in turn creating hyperprolactinemia. The mesocorticllimbic pathway transmits dopamine to the prefrontal cortex and midbrain, and disorders in this pathway are associated with thought disorders.
4-05. A 72-year-old man with a comorbid history of uncontrolled hypertension is referred to the PMHNP for dementia secondary to depression. The mental status exam reveals difficulty speaking fluently, inappropriate and confabulated responses, incongruent affect, and poor judgment. A cerebrovascular accident in which region of the brain would better explain these symptoms?
A. Parietal lobe
B. Wernicke’s area
C. Broca’s area
D. Occipital lobe
Answer C. Broca’s area, located in the dominant hemisphere of the brain, is associated with speech production and fluency; dysfunctions in this area are associated with incongruent affect, decreased motivation, impaired judgment and attention, and confabulation. Wernicke’s area is associated with comprehension. The parietal lobe is the area of sensory integration, spatial awareness, and conscious awareness of the opposite side of the body. The occipital lobe is associated with the visual cortex and receives signals through the optic tract and interprets color, form, and movement of visually perceived objects.
4-06. Which region of the brain is associated with sensory integration, spatial awareness, and conscious awareness of the opposite side of the body?
A. Occipital lobe
B. Wernicke’s area
C. Broca’s area
D. Parietal lobe
Answer D. The parietal lobe is the area of sensory integration, spatial awareness, and conscious awareness of the opposite side of the body. The occipital lobe is associated with the visual cortex signals through the optic tract and interprets color, form, and movement of visually perceived objects. Broca’s area, located in the dominant hemisphere of the brain, is associated with speech production and fluency; dysfunctions in this area are associated with incongruent affect, decreased motivation, impaired judgment and attention, and confabulation. Wernicke’s area is associated with comprehension.
4-07. A patient appears internally preoccupied and responding to auditory hallucination. This patient may have a pathology affecting which lobe of the brain?
A. Temporal lobe
B. Occipital lobe
C. Parietal lobe
D. Cerebellum
Answer A. The temporal lobe is the primary auditory area of the brain. The occipital lobe is associated with the visual cortex signals through the optic tract and interprets color, form, and movement of visually perceived objects. The parietal lobe is the area of sensory integration, spatial awareness, and conscious awareness of the opposite side of the body.
4-08. A patient presents with atypical symptoms of psychosis that include inconsistent auditory hallucinations without disorganization. What is the best definition of an EEG and its utility?
A. Radiologic test utilizing magnets and radio waves to provide a detailed image of the structures of the brain
B. Electrophysiologic test that depicts electrical activity on the surface of the brain
C. A serum test to indicate nutritional deficiencies that inhibit the formation of neurotransmitters
D. A neuropsychological test to distinguish psychosis from a psychosomatic illness.
Answer B. An EEG is an electrophysiological test that depicts electrical activities as various waveforms from the surface of the brain; these may indicate an ectopic stimulus that may be responsible for neurological symptoms. An MRI is a nonradioactive imaging test that uses magnets and radio waves to provide a detailed image of the structures of the brain. No serum test exists to distinguish nutritional deficiencies responsible for the formation of neurotransmitters. A genetic test exists for methyl-tetrahydrofolate reductase (MTHFR) expression, indicating the ability to convert folic acid to L-methylfolate. Neuropsychological tests are psychometrically sound measurement tools in which the scores are computed to support evidence for a particular diagnosis and focused on the functional correlation associated with an area of the brain.
4-09. Neurons that only travel from the body to the brain are called:
A. Sensory neurons
B. Motor neurons
C. Interneurons
D. Astrocytes
Answer A. Sensory neurons are afferent fibers transmitting information from stimuli from the body to the brain. Motor neurons are efferent fibers conducting impulses from the brain to the body. Interneurons are a neuronal circuit allowing for communication between motor and sensory neurons, especially within the reflex arc. Astrocytes are star-shaped cells; they provide nutrients and maintain the extracellular ionic balance involved in growth and repair of nerve cells to maintain the blood-brain barrier.
4-10. The part of the neuron that takes in and receives messages is called:
A. Dendrite
B. Axon
C. Synapse
D. Node of Ranvier
Answer A. Dendrites transmit information to the soma. Axons transmit information from the soma. A synapse is the gap between two nerve cells. The node of Ranvier is a gap between the myelin sheaths of Schwann cells and the uninsulated ion channel.
4-11. What is the fatty layer that prevents interference and increases the speed of impulse transmission along the axon?
A. Dendrite
B. Axon
C. Synapse
D. Myelin sheath
Answer D. The myelin sheath is the fatty layer that prevents interference and increases the speed of impulse transmission along the axon. Dendrites transmit information to the soma. Axons transmit information from the soma. A synapse is the gap between two nerve cells.
4-12. Which of the following receives messages and conducts impulses to the soma?
A. Dendrite
B. Axon
C. Synapse
D. Myelin sheath
Answer A. Dendrites transmit information to the soma. Axons transmit information from the soma. Synapses are the gap between two nerve cells. The myelin sheath is the fatty layer that prevents interference and increases speed of impulse transmission along the axon.
4-13. The junction between two neurons is known as the:
A. Dendrite
B. Node of Ranvier
C. Synapse
D. Soma
Answer C. A synapse is the gap between two nerve cells. Dendrites transmit information to the soma. The node of Ranvier is a gap between the myelin sheaths of Schwann cells and the uninsulated ion channels. The soma is the body of the nerve cell containing the nucleus.
4-14. When explaining the means by which neurotransmitters relate to mental illness, a patient ask, “What is a neurotransmitter?” The best answer is:
A. A fatty layer covering the axon
B. A chemical messenger
C. A nerve cell
D. The space between nerve cells
Answer B. A neurotransmitter is a chemical messenger, created from dietary substrates, that transmits information between neurons. The myelin sheath is the fatty layer that prevents interference and increases the speed of impulse transmission along the axon. The soma is the body of the nerve cell containing the nucleus. Synapse is the gap between two nerve cells.
4-15. A patient is diagnosed with major depressive disorder with the most prominent feature of apathy, anhedonia, and isolation. Which neurotransmitter is associated with the symptoms?
A. Serotonin
B. Gamma-aminobenzoic acid (GABA)
C. Dopamine
D. Cannabinoids
Answer C. Dopamine increases a sense of well-being and satisfaction; it is the primary driver of the reward system. Serotonin reduces pain perception and enhances satiety. Gamma-aminobenzoic acid (GABA) is primarily inhibitory; it increases sleepiness and reduces anxiety, memory, and alertness. Cannabinoids increase hunger and reduce motivation and sex drive.
4-16. The PMHNP prescribes fluoxetine to treat depression. Which neurotransmitter will be made readily available in the synapse?
A. Dopamine
B. Serotonin
C. Gamma-aminobenzoic acid (GABA)
D. Acetylcholine
Answer B. Serotonin reduces pain perception and enhances satiety. Gamma-aminobenzoic acid (GABA) is primarily inhibitory; it increases sleepiness and reduces anxiety, memory, and alertness. Dopamine increases a sense of well-being and satisfaction; it is the primary driver of the reward system. Acetylcholine increases heart rate, muscle contractions, salivation, and sweating; it is prescribed to enhance memory.
4-17. The PMHNP in the ED is evaluating a 68-year-old woman who was recently diagnosed with mild cognitive impairment and cannot remember the name of the medication that she has recently started 1 week ago. The patient is complaining of increased heart rate, sweating, and muscle spasms progressively worsening for the last 3 days. Which neurotransmitter is associated with this adverse effect?
A. Serotonin
B. Acetylcholine
C. Gamma-aminobenzoic acid (GABA)
D. Dopamine
Answer B. Acetylcholine increases heart rate, muscle contractions, salivation, and sweating; it is prescribed to enhance memory. Serotonin reduces pain perception and enhances satiety. Gamma-aminobenzoic acid (GABA) is primarily inhibitory; it increases sleepiness and reduces anxiety, memory, and alertness. Dopamine increases a sense of well-being and satisfaction; it is the primary driver of the reward system.
4-18. Which of the following neurotransmitters exert inhibition over neuronal function?
A. Acetylcholine
B. Cannabinoids
C. Gamma-aminobenzoic acid (GABA)
D. Dopamine
Answer C. Gamma-aminobenzoic acid (GABA) is primarily inhibitory; it increases sleepiness and reduces anxiety, memory, and alertness. Acetylcholine increases heart rate, muscle contractions, salivation, and sweating; it is prescribed to enhance memory. Cannabinoids increase hunger and reduce motivation and sex drive. Dopamine increases a sense of well-being and satisfaction; it is the primary driver of the reward system.
4-19. A patient with alcohol use disorder is admitted for detox in anticipation of admission to an inpatient rehabilitation facility. The patient takes metoprolol for hypertension, which he has forgotten to take today, and his last drink was 72 hours ago. Which tool should the PMHNP use to evaluate for symptoms of withdrawal?
A. Hamilton Rating Scale for Depression
B. Clinical Institute Withdrawal Assessment
C. Vital signs
D. Quick Inventory of Depressive Symptomatology
Answer B. Clinical Institute Withdrawal Assessment (CIWA) is a nine-item, clinician-observed rating scale to quantify the severity of alcohol withdrawal. The Hamilton Rating Scale for Depression is a self-report scale to measure depressive symptoms, and is commonly used at baseline and to measure treatment progress. Vital signs are a nonspecific measure of hemodynamic stability. The Quick Inventory of Depressive Symptomatology is a self-report survey assessing the severity of depressive symptoms.
4-20. A 45-year-old man presents for a follow-up visit after starting on an SSRI 2 weeks ago for major depressive disorder. Which assessment tool would be the best indicator of progress?
A. Geriatric Depression Scale
B. The Young Mania Rating Scale
C. Yale-Brown Obsessive Compulsive Scale
D. Quick Inventory of Depressive Symptomatology
Answer D. Quick Inventory of Depressive Symptomatology is a self-report survey assessing the severity of depressive symptoms. The Geriatric Depression Scale is a self-report binary questionnaire used to measure depressive symptoms (concentration, self-image, agitation, losses, motivation, obsessive traits) in older adults.
5-01. The PMHNP knows that the study of how the body absorbs, distributes, metabolizes, and excretes a medication is known as:
A. Breakdown
B. Pharmacokinetics
C. Pharmacodynamics
D. First-pass effect
Answer B. Pharmacokinetics is the study of what the body does to a drug concerning absorption, distribution, metabolism, and excretion (ADME). The breakdown is one aspect of metabolism in which the drug is prepared for bodily excretion. Pharmacodynamics is the study of the relationship between drug concentration and effect on the body. First-pass effect is the uptake and conversion of the drug in the liver after enteric absorption.
5-02. After a drug is absorbed the substrate binds to protein for transport. Which portion of the drug is available for therapeutic effects?
A. Bound
B. Unbound
C. Metabolized
D. Excreted
Answer B. Unbound drug is the portion of the drug available for therapeutic effects. Bound is the part of the drug that binds to protein and fat in preparation for excretion. Metabolization is the process in which a drug is prepared for excretion. Excretion is the process in which a substance leaves the body.
5-03. A 19-year-old male is referred to the PMHNP by the student health office for alcohol use disorder. The patient states that on one occasion he passed out much sooner than he usually would with far less than he would usually drink. Upon further interview, the patient reveals the time he passed out was during a fraternity hazing in which he was butt-chugging (receiving a beer and vodka enema). What pharmacokinetic process was bypassed by this rectal administration route?
A. Excretion
B. Absorption
C. Distribution
D. First-pass effect
Answer D. The first-pass effect is the precirculation process of uptake and conversion by which a substrate is significantly reduced through the CYP450 pathway. Nonenteric routes of administration bypass this effect. Excretion is the process by which substances leave the body. Absorption is the movement of the drug into the bloodstream. Distribution is the movement of the drug through the bloodstream to target receptors where protein binding occurs.
5-04. The PMHNP is monitoring a serum drug level for a medication with a 24-hour half-life. How many hours will it take to reach steady state?
A. 48 hours
B. 72 hours
C. 96 hours
D. 120 hours
Answer D. 120 hours; steady state is achieved in five half-lives of the medication (5 × 24 = 120).
5-05. A patient with schizophrenia was discharged from the hospital on olanzapine 5 mg twice a day. He immediately resumed smoking cigarettes and escalated to one pack per day. Upon presenting for his 1-week follow-up appointment, the patient reports he is having trouble sleeping and the voices have started to return. Which of the following actions should the PMHNP take?
A. Send the patient to the ED for stabilization
B. Change to another antipsychotic medication, and refer to a psychiatrist
C. Increase his olanzapine, and schedule a follow-up visit in 2 days
D. Tell him to stop smoking and give him a nicotine patch
Answer C. Increase his olanzapine and schedule a follow up visit because the patient’s symptoms are no longer controlled and smoking is a known inducer of the CYP450 pathway. The patient has not indicated a threat of harm to self or others; changing to another antipsychotic medication requires retitration and is not indicated. Telling a patient to stop smoking may trigger a psychological paradox and can erode therapeutic alliance.
5-06. A patient who has been stable on quetiapine (Seroquel XR) for 3 months has decided to start to drink grapefruit juice twice daily because she has heard it helps with weight loss. She calls to report that since her new diet she has been feeling fatigue and difficulty waking up in the morning. What is the best response by the PMHNP?
A. “I will put in a new prescription for a lower dose of your medication.”
B. “Stop drinking the grapefruit juice, and schedule an appointment to discuss the matter further.”
C. “I will prescribe you a stimulant, and see you in 2 weeks.”
D. “You should make an appointment with your primary care provider for evaluation.”
Answer B. Stop drinking the grapefruit juice and schedule an appointment to discuss weight gain/weight loss. Grapefruit is a known inhibitor of the CYP450 pathway; stopping this will reduce the sedation over time. It is not appropriate to prescribe this patient a stimulant as the cause has not been determined. Deferring the patient to primary care is not suitable as there is a potential psychotropic drug interaction.
5-07. A 79-year-old female with no past psychiatric history is admitted with new onset auditory hallucinations. The patient states her most bothersome symptom is the voices, which have kept her from sleeping through the night for the last 3 weeks, and as a result, she is tired and irritable. Which medication should the PMHNP order for bedtime for sleep?
A. Olanzapine 2.5 mg PO
B. Lorazepam 1 mg PO
C. Chlorpromazine 50 mg PO
D. Sertraline 50 mg PO
Answer A. Olanzapine is an atypical antipsychotic that can help the patient sleep. Because of the patient’s age, it is essential to start at a low dose and slowly titrate as tolerated. Benzodiazepines are not preferred in the elderly as they increase risk especially at higher starting doses. 92Chlorpromazine potentiates many anticholinergic effects particularly at high starting doses. Sertraline is not sedating, and the dose should be started lower in the elderly.
5-08. A 16-year-old male with a history of smoking cigarettes and marijuana is being evaluated at the request of the school and parents. The child states that the reason he smokes is it “helps calm me down” as he is anxious all the time. Which of the following medications would be the best choice?
A. Fluoxetine 20 mg PO daily
B. Clonazepam 0.5 mg PO daily
C. Vortioxetine 20 mg daily
D. Citalopram 5 mg PO daily
Answer A. Fluoxetine is a selective serotonin reuptake inhibitor (SSRI) with a long half-life, approved in children for depression and anxiety. Children’s dosages should be started at the usual (not lower dose) given their faster metabolisms. Clonazepam is a benzodiazepine and should be avoided given the high potential for abuse and addiction in a patient who is demonstrating illegal behavior. Vortioxetine is not approved in children, and citalopram, while approved, is at too low a dose for starting in children.
5-09. A patient has had a pharmacogenomic test, which reveals he is an extensive metabolizer on CYP450, 1A2, 2D6, and 3A4. The PMHNP should prescribe:
A. A lower than usual dose
B. The usual dose
C. A higher than usual dose
D. A different medication
Answer B. Extensive metabolizers are normal, and medications should be prescribed according to the prescriber’s information. Poor metabolizers have two defective genes and require lower doses of medicine and a long titration schedule and may not be able to achieve therapeutic effect. Slow metabolizers need lower doses of medicine as well. Ultra-rapid metabolizers need higher doses of drug. Changing the drug would not be appropriate as extensive metabolizers are normal.
5-10. A woman in her 20th week of pregnancy has been resumed on lithium for bipolar disorder. The PMHNP knows that the patient may become subtherapeutic despite taking the medication as prescribed due to:
A. Increased blood volume
B. Increased fetal metabolism
C. Reduced muscle mass
D. Reduced blood volume
Answer A. Increased blood volume occurs as pregnancy progresses and patients may need higher doses of medication to maintain concentration effects. Fetal metabolism has no impact on maternal metabolism, but caution is exercised for potential adverse effects to the fetus. Pregnant women do not have decreased muscle mass or reduced blood volume.
5-11. The PMHNP knows the relationship between drug concentration and effect on the body is known as:
A. Pharmacology
B. Pharmacokinetics
C. Pharmacodynamics
D. Physiology
Answer C. Pharmacodynamics is the study of the relationship between drug concentration and effect on the body. Pharmacology is the study of medication uses, effects, and modes of action. Pharmacokinetics is concerned with the body’s effect on the drug concerning absorption, distribution, metabolism, and excretion. Physiology is a biological branch concerned with normal functions of an organism.
5-12. A patient who had previously reported feeling much better on her antidepressant suddenly says the “drug seems to have stopped working. This is known as:
A. Tolerance
B. Tachyphylaxis
C. Side effect
D. Adverse effect
Answer B. Tachyphylaxis is the rapidly diminishing responsiveness to increasing doses of the medication, also known as the poop-out effect. Tolerance is the process of becoming desensitized and therefore less responsive to a particular dose of medication over time, thereby requiring an increase.
5-13. A medication that works by receptor activation to produce a biological response is an:
A. Agonist
B. Enzyme inhibitor
C. Inverse agonist
D. Antagonist
Answer A. An agonist is a medication that works by receptor activation to produce a biological effect. An enzyme inhibitor slows the catalytic action of the enzyme and allows neurotransmitters to remain in circulation. An inverse agonist is a medication that binds to the same receptor as an agonist but induces an opposite biological response. An antagonist is a medication that blocks a receptor and inhibits a biological response, even from endogenous agonists.
5-14. A drug that binds to a receptor but does not fully activate it, producing a muted biological response, is a(n):
A. Agonist
B. Enzyme inhibitor
C. Partial agonist
D. Antagonist
Answer C. Partial agonists bind to a receptor but do not fully activate it, and produce a muted biological response. An agonist is a medication that works by receptor activation to produce a biological effect. An enzyme inhibitor slows the catalytic action of the enzyme, and allows neurotransmitters to remain in circulation. An antagonist is a medication that blocks a receptor and inhibits a biological response, even from endogenous agonists.
5-15. A drug that binds to the same receptor as an agonist, but induces an opposite biological response, is a(n):
A. Antagonist
B. Inverse agonist
C. Agonist
D. Partial agonist
Answer B. An inverse agonist is a medication that binds to the same receptor as an agonist but induces an opposite biological response. An antagonist is a medication that blocks a receptor and inhibits a biological response, even from endogenous agonists. An agonist is a medication that works by receptor activation to produce a biological effect. Partial agonists bind to a receptor but do not fully activate it, and produce a muted biological response.
5-16. A drug that blocks receptors, inhibiting a biological response, is known as a(n):
A. Partial agonist
B. Inverse agonist
C. Antagonist
D. Agonist
Answer C. An antagonist is a medication that blocks a receptor to inhibit a biological response even from endogenous agonists. Partial agonists bind to a receptor but do not fully activate it, and produce a muted biological response. An inverse agonist is a medication that binds to the same receptor as an agonist but induces an opposite biological response. An agonist is a medication that works by receptor activation and produces a biological effect.
5-17. One of the most common electrolyte abnormalities associated with the use of psychotropic medication is:
A. Hyperkalemia
B. Hypercalcemia
C. Hyponatremia
D. Hypernatremia
Answer C. Hyponatremia is the most common electrolyte abnormality associated with psychotropic medication; the condition is called drug-induced syndrome of inappropriate 93antidiuretic hormone secretion. Hyperkalemia is related to renal insufficiency, which can be acute or chronic. Hypercalcemia is associated with neoplastic disorders and abnormal bone metabolism and resorption. Hypernatremia is related to a water deficit.
5-18. A patient who has been stable on his long-acting injectable medication tells the PMHNP that he would like to quit smoking. What is the best response?
A. “Good for you, I can prescribe you a nicotine patch to help.”
B. “Let us make an appointment as your medication dose may need to be lowered.”
C. “I will refer you to a smoking cessation support group.”
D. “I am glad you have decided to quit smoking.”
Answer B. Making an appointment with the patient for psychoeducation and careful medication adjustment is most appropriate as the patient may become more sedated when the inducing properties of cigarette smoke are less present in the system. Verbal reinforcement and prescribing a medication sight unseen is not the preferred response. Referring the patient to smoking cessation, while helpful, does not address the potential drug–drug interactions. Expressing encouragement for healthy behaviors would be insufficient in this case.
5-19. A patient has been taking valproic acid for mood stabilization from a manic episode but is still not sleeping through the night. His last drug level was 50 mcg/mL. The PMHNP notices the patient seems disoriented to time and is flapping his wrists. What should the PMHNP do next?
A. Give the patient lactulose 20 g PO
B. Draw an ammonia level
C. Increase the dose of valproic acid
D. Augment with an atypical antipsychotic
Answer B. Patients can develop encephalopathy due to hyperammonemia when taking valproic acid. It is essential to check the ammonia level before treating the patient. Increasing the dose of medication would worsen the situation, and adding another agent does not address the underlying problem.
5-20. A patient who has been taking paroxetine for 3 years wants to come off due to sexual side effects. He had gradually tried to wean the dose on his own but noticed feelings of dizziness, fatigue, headaches, anxiety, and electric-like shocks in his head. These findings are most consistent with SSRI discontinuation. What would be a reasonable action for the PMHNP to take?
A. Reassure the patient that the symptoms will go away eventually
B. Prescribe the patient fluoxetine for 1 week
C. Have the patient resume the paroxetine and prescribe the patient a PD5-inhibitor
D. Start the patient on an antipsychotic for his somatic delusions
Answer B. Prescribing the patient fluoxetine (a long half-life selective serotonin reuptake inhibitors [SSRI]) will reduce the discontinuation syndrome symptoms. Reassuring the patient may provide some psychological comfort, but it does not address the physical distress of the patient. Resuming the medication and prescribing a second agent for sexual dysfunction does not solve the present problem. There is no evidence that the patient is having a somatic delusion as discontinuation syndrome is well documented in the literature associated with paroxetine.
6-01. A systematic inquiry that elaborates on the chief complaint while eliciting pertinent positive and pertinent negative factors from the history is known as:
A. The mental status exam
B. Review of systems
C. Physical exam
D. Diagnostic formulation
Answer B. The review of systems describes the systematic inquiry that elaborates on the chief complaint and elicits pertinent positive and pertinent negative factors from the health history. The mental status exam is a neuropsychological test used to classify observations associated with a psychiatric disorder. The physical exam is an organized approach to gather objective data from the patient’s body regions and organ systems. The diagnostic formulation is an articulate summary of biopsychosocial findings that account for the patient’s chief complaint.
6-02. The mental status exam includes all of the following except:
A. Appearance and behavior
B. Mood and affect
C. Cranial nerves and reflexes
D. Insight and judgment
Answer C. Cranial nerves and reflexes are complements of the physical exam. The mental status exam includes assessments of appearance, behavior, mood, affect, insight, and judgment.
6-03. A 72-year-old female presents to the emergency department for altered mental status. Her urine analysis is negative, and her labs are normal. She has been referred to psychiatry for dementia. The patient’s husband reports increasing forgetfulness over the last 6 months, walking as if her feet are stuck to the ground, and most recently episodes of urinary incontinence. Based on the signs and symptoms which of the following diagnostic tests should the PMHNP order to rule out an underlying condition?
A. Lumbar puncture
B. Noncontrast head CT scan
C. Arterial blood gas
D. Electroencephalogram
Answer B. A noncontrast head CT scan is helpful in excluding the diagnosis of normal pressure hydrocephalus, which presents with an abnormal (magnetic) gait, urinary incontinence, and cognitive decline. Lumbar puncture is used to diagnose infectious, autoimmune issues, and paraneoplastic syndromes associated with the nervous system. Arterial blood gases are used to measure concentrations of atmospheric gases in patients with respiratory distress. An electroencephalogram is used to measure brain wave activity in people with suspected seizure and sleep disorders.
6-04. The condition in which the patient is unable to discriminate between objects based on touching is known as:
A. Agraphesthesia
B. Choreiform
C. Astereognosis
D. Dysdiadochokinesia
Answer C. Astereognosis is the condition in which the patient is unable to discriminate between objects based on touching. Choreiform movements describe repetitive and rapid jerking and involuntary coordinated movements. Agraphesthesia is the inability to recognize letters or numbers drawn on the palm with a pointed object. Dysdiadochokinesia is the inability to perform rapidly alternating hand movements.
6-05. The PMHNP is conducting a neurological examination on a patient with altered mental status. The inability to perform rapidly alternating hand movements is known as:
A. Agraphesthesia
B. Choreiform
C. Astereognosis
D. Dysdiadochokinesia
Answer D. Dysdiadochokinesia is the inability to perform rapidly alternating hand movements. Agraphesthesia is the inability to recognize letters or numbers drawn on the palm with a pointed object. Choreiform movements describe repetitive and rapid jerking and involuntary coordinated movements. Astereognosis is the condition in which the patient is unable to discriminate between objects based on touching.
6-06. The PMHNP is observing a patient making repetitive and rapid jerking involuntary coordinated movements. In documenting the findings of the mental status exam, which of the following concisely describes this behavior?
A. Agraphesthesia
B. Choreiform
C. Astereognosis
D. Dysdiadochokinesia
Answer B. Choreiform movements describe repetitive and rapid jerking and involuntary coordinated movements. Agraphesthesia is the inability to recognize letters or numbers drawn on the palm with a pointed object. Astereognosis is the condition in which the patient is unable to discriminate between objects based on touching. Dysdiadochokinesia is the inability to perform rapidly alternating hand movements.
6-07. The PMHNP is supervising a nurse practitioner student. When presenting the case, the student notes that the patient has an inability to recognize letters or numbers drawn on the palm with a pointed object. This sign is called:
A. Agraphesthesia
B. Choreiform
C. Astereognosis
D. Dysdiadochokinesia
Answer A. Agraphesthesia is the inability to recognize letters or numbers drawn on the palm with a pointed object. Choreiform movements describe repetitive and rapid jerking and involuntary coordinated movements. Astereognosis is the condition in which the patient is unable to discriminate between objects based on touching. Dysdiadochokinesia is the inability to perform rapidly alternating hand movements.
6-08. A patient is started on lamotrigine for bipolar depression. The patient complained of a rash comprised of blistering skin and mouth sores after a dose increase. The blood work reveals normal renal and liver function. Which type of drug rash is most likely?
A. DRESS syndrome
B. Stevens–Johnson syndrome
C. Psoriasis
D. Atopic dermatitis
Answer B. Stevens–Johnson syndrome is a specific, life-threatening drug rash characterized by blisters giving way to dermis and hives on the lining of any mucous membranes. Drug rash with eosinophilia and systemic symptoms (DRESS) syndrome is associated with a rise in eosinophils, liver enzymes, renal insufficiency, and positive cardiac enzymes. Psoriasis is characterized by scaly patches and erythema, and can exacerbate with streptococcal and viral infections, and exposure to immunizations and antimalarial drugs. Atopic dermatitis is a common inflammatory skin disease which varies in presentation from dry skin scales to blisters and fissuring.
6-09. In observing the appearance of the patient with depression and a history of systemic lupus erythematosus, the PMHNP notices a lesion on the neck of the patient about the size of a pencil eraser, flush with the skin, uniformly colored, with discrete borders and consistent texture. The PMHNP should:
A. Refer the patient to dermatology
B. Refer the patient to rheumatology
C. Refer the patient to oncology
D. Watch and wait
Answer D. Watching and waiting is appropriate in this scenario, as long as moles are <6 mm, flush with the skin, symmetrical in size, shape, texture, and color, and have regular borders. The lesion does not have characteristics consistent with cancerous lesions or autoimmune pathology.
6-10. A 35-year-old man diagnosed with bipolar disorder mentions to his PMHNP that he had felt a lump on his left testicle. He is concerned he has not had a physical exam in 10 years. What is the best action by the PMHNP?
A. Perform a urogenital exam and order an ultrasound
B. Explore the reason for not having had a physical in so long
C. Refer him to his primary care provider
D. Refer him to urology
Answer B. Exploring the reasons for not having had a physical exam in 10 years will help provide the necessary information for anticipatory guidance as the patient undergoes diagnostic investigation. Urogenital exams are generally not performed by psychiatric providers. Referring the patient to his primary care provider or urology would be appropriate but without further assessment specific barriers to accessing care remain unknown.
6-11. A 26-year-old female is diagnosed with psychosis not otherwise specified (NOS) and substance use disorder. Which of the following lab tests should be ordered prior to starting antipsychotic medications?
A. Hemoglobin A1C, lipid profile, EKG
B. Urinalysis, urine pregnancy test
C. Amylase, lipase, liver function test
D. Cardiac enzymes, brain naturetic peptide
Answer A. Antipsychotic medications are known to accelerate the onset of type 2 diabetes mellitus and dyslipidemia, and prolong the QTc interval; baseline evaluation provides a frame of reference for ongoing treatment and monitoring. Urinalysis and a pregnancy test would not preclude or be affected by antipsychotic medications. Pancreatic and liver function tests are not known to be affected by antipsychotic medications. Cardiac enzymes and brain naturetic peptide are used to evaluate cardiac function, which can be affected by clozapine-induced myocarditis, but require clinical correlation to confirm suspicion, and should not be ordered de novo.
6-12. A 57-year-old man was evaluated in the ED for altered mental status. His head CT scan is negative, white blood cells are 6,000, urinalysis negative for infection with positive ketones, serum glucose is 120 mg/dL, sodium is 148, blood urea nitrogen 22, creatinine 1.8, and cardiac enzymes are negative. Neurology has cleared him for an acute event, and the ED physician has cleared him for psychiatry. The PMHNP evaluating him obtained collateral information that the patient has been in bed for 3 days, and not eating or drinking because the voices tell him the food is bad and the water is contaminated. What should the nurse practitioner do first?
A. Order 1 L normal saline at 100 mL/hr
B. Order Haldol 5 mg PO × 1
C. Order a lumbar puncture
D. Order hemoglobin A1C and lipid profile
Answer A. The clinical history and lab findings are suggestive of dehydration, which can account for delirium; this must be excluded prior to diagnosing a psychiatric disorder. Haldol is an antipsychotic medication with sedating effects, which would cloud the diagnostic formulation. Lumbar puncture is ordered to rule out infections, and autoimmune and paraneoplastic syndromes with clinical suspicion. Hemoglobin A1C and lipid profiles are ordered at baseline before starting antipsychotic therapy for continued use.
6-13. A 30-year-old man with history of alcohol use disorder is being evaluated for altered mental status. He has a negative blood alcohol level, aspartate transaminase 100, alanine transaminase 90, hemoglobin 10, hematocrit 30, negative stool for occult blood, negative head CT scan. What further tests should the PMHNP order before diagnosing psychosis not otherwise specified?
A. EKG
B. Lipid profile
C. Hemoglobin A1C
D. Serum ceruloplasmin
Answer D. Serum ceruloplasmin (copper-carrying protein) is decreased in most instances of Wilson’s disease in which there is a decreased ability to incorporate copper into apoceruloplasmin, resulting in increased free copper in plasma, liver, and brain tissue. EKG, hemoglobin A1C, and lipid profile do not provide an explanation for the signs and symptoms presented.
6-14. The Consultation Liaison PMHNP is asked to evaluate an 88-year-old woman admitted to medicine with community acquired pneumonia for capacity regarding intubation. On evaluation the patient is inattentive and easily distracted, with waxing and waning sensorium. The patient was recently started on broad spectrum IV antibiotics. What is the most likely diagnosis?
A. Depression
B. Dementia
C. Delirium
D. Psychosis
Answer C. Delirium is characterized by an acute onset waxing and waning sensorium, inattentiveness, and distractibility, and can be caused by infection, new environment, and medications. Depression is a psychiatric disorder that must have symptoms present for at least 2 weeks. Dementia is a disorder characterized by chronically impaired cognition and may include changes in personality over time. Psychosis is characterized by disorganized thinking, alterations in perception of reality, and an inability to care for oneself.
6-15. When evaluating a new 40-year-old, 4-week postpartum woman complaining of fatigue, irritability, dry skin, and poor sleep quality for mood disorder, which of the following diagnostic tests should be ordered to aid in formulating a diagnosis?
A. Hemoglobin A1C
B. EKG
C. Thyroid-stimulating hormone (TSH) and free T4
D. Urine pregnancy test
Answer C. TSH and free T4 can be affected in the postpartum period and cause signs and symptoms of thyroiditis, which include fatigue and irritability. Mood disorder is a diagnosis of exclusion, which can have similar symptoms to hypothyroidism. Hemoglobin A1C, EKG, and urine pregnancy tests are not indicated as they will not account for the presenting symptoms and clinical scenario.
6-16. A 40-year-old man who has been on olanzapine for schizophrenia has a lipid profile indicating total cholesterol 240 mg/dL, and low-density lipoprotein (LDL) 200 mg/dL. Which of the following is recommended?
A. Lifestyle modification
B. Statin therapy
C. Serial EKGs
D. Transthoracic echocardiogram
Answer B. 112Statin therapy is indicated for patients with LDL >70 mg/dL with diabetes, patients with known atherosclerotic heart disease (ASCVD) and high total cholesterol, patients with an LDL >190 mg/dL, and patients with diabetes between the ages of 45 to 75.
6-17. A 24-year-old man has been stable on lithium 1,800 mg daily for 2 years. His most recent serum lithium level is 2.0. The patient has no signs or symptoms of toxicity. Which of the following is the best action by the PMHNP?
A. Tell the patient to stop his medications immediately
B. Encourage the patient to drink more water
C. Repeat the lithium level and ensure it is a trough
D. Send the patient to the ED
Answer C. Repeat the lithium level and ensure it is a trough as the patient has not had any dosage changes and has no signs or symptoms of toxicity. Encouraging the patient to drink more water assumes the elevated level is correct. Telling the patient to stop the medication may cause his level to drop below therapeutic range and require retitration. Sending the patient to the ED is viable, but the blood draw may not be at a time when the lithium is at the trough.
6-18. A 19-year-old female is evaluated in the ED for admission to an eating disorders unit and has a history of bulimia nervosa. Which of the following diagnostic tests should be ordered?
A. Electrolytes and renal function
B. Liver and pancreatic function
C. Lipid profile and hemoglobin A1C
D. Urinalysis and urine pregnancy test
Answer A. Electrolytes and renal function will help determine if the patient is medically stable to be in an eating disorders unit. Liver and pancreatic function tests are not acutely affected by bulimia nervosa. Lipid profile and hemoglobin A1C is not affected acutely by bulimia nervosa. Regardless of the results of the urinalysis and pregnancy test, the patient may still be treated for the eating disorder.
6-19. A 26-year-old man has been on an antipsychotic. He requires further titration for control of symptoms. Which of the following tests should be repeated?
A. EKG
B. Liver function test
C. Urinalysis
D. Head CT scan
Answer A. The EKG test should be repeated, as antipsychotics are known to prolong the QTc (except for aripiprazole). Liver function tests, urinalysis, and head CT scans are not affected by antipsychotic medication titration.
6-20. Which of the following is an indication for monitoring of serum drug levels?
A. Smoking cessation
B. New diagnosis of diabetes
C. Resolution of symptoms
D. New diagnosis of hyperlipidemia
Answer A. Smoking cessation is an indication for monitoring serum drug levels because smoking is a known inducer of the CYP450 pathway. The patient may have higher than normal/toxic serum drug levels as a result. The new diagnosis of diabetes or hyperlipidemia or resolution of symptoms are not indications for testing serum drug levels if the medications are prescribed within the recommended timeframe.
7-01. A 61-year-old married woman who has been sober from alcohol use disorder for 25 years relapsed 3 weeks ago. She reports drinking 1.5 L of vodka and half a liter of scotch daily. On exam she is emotionally labile ranging from irritable to crying, slurring her words, and exhibits a bilateral upper extremity fine tremor. Vitals: blood pressure 150/100, heart rate 110, respiratory rate 18, SpO2 98% room air. She reports feeling nauseous, sweating, and moderately anxious. What is the primary concern for this patient?
A. Alcohol use disorder
B. Generalized anxiety disorder
C. Alcohol withdrawal
D. Delirium tremens
Answer C. Alcohol withdrawal symptoms include emotional lability, bilateral upper extremity tremor, hypertension, tachycardia, nausea, sweating, and anxiety. Alcohol use disorder is characterized by a pattern of use with significant impairment with two or more Diagnostic and Statistical Manual of Mental Disorders, Fifth Edition (DSM-5) criteria over the previous 12-month period. Generalized anxiety disorder is characterized by excessive worrying lasting at least 6 months. Delirium tremens is a severe form of alcohol withdrawal marked by global confusion, audiovisual hallucinations, fever, hypertension, and diaphoresis, which occur within 3 to 10 days of the last alcoholic drink.
7-02. A patient who has required increasing amounts of medication to achieve the desired effect and develops withdrawal symptoms when dosing is delayed is said to be:
A. Dependent
B. Tolerant
C. Addicted
D. In withdrawal
Answer A. Dependence is characterized by the repeated use of a substance for physical needs leading to increased tolerance and, when discontinued, results in physical withdrawal symptoms. Tolerance is a condition in which repeated administration of the dosage causes a decreased effect despite increasingly larger doses, which are required to obtain the effect observed in the initial dosage. Addiction is psychological dependence leading to persistent and increased use of the substance or behavior and when discontinued causes distress and urges to resume use 125despite adverse consequences. Withdrawal is a substance-specific cluster of signs and symptoms that occur when reducing or stopping the consistent use of the substance.
7-03. A patient is complaining of chronic back pain and requests a refill of his oxycodone prescription because the pain is so bad he becomes suicidal. As a result, his primary care provider referred him to the PMHNP. The patient states he can tell if he is late taking his next dose by even 1 hour as he becomes very anxious, restless, irritable, and begins sweating. He states that he is so worried about having pain attacks that much of his day is spent waiting to take the next dose. When he runs out of his medication between visits he buys some from his friend. Which of the following best describes the patient’s condition?
A. Dependence
B. Addiction
C. Tolerance
D. Withdrawal
Answer B. Addiction is psychological dependence leading to persistent and increased use of the substance or behavior and, when discontinued, causes distress and urges to resume use despite adverse consequences. Dependence is characterized by the repeated use of a substance for physical needs leading to increased tolerance and, when discontinued, results in physical withdrawal symptoms. Tolerance is a condition in which repeated administration of the dosage causes a decreased effect despite increasingly larger doses, which are required to obtain the effect observed in the initial dosage. Withdrawal is a substance-specific cluster of signs and symptoms that occur when reducing or stopping the consistent use of the substance.
7-04. A 30-year-old male who was started on sertraline for generalized anxiety disorder was also started on lorazepam 1 mg for sleep during the first 2 weeks as he was adjusting to the medication. At the 2-week follow-up visit his psychiatrist increased the sertraline, but the dose of lorazepam does not seem to be helping with sleep and the patient is requesting a higher dose. Which of the following best describes the patient’s condition?
A. Dependence
B. Addiction
C. Tolerance
D. Withdrawal
Answer C. Tolerance is a condition in which repeated administration of the dosage causes a decreased effect despite increasingly larger doses which are required to obtain the effect observed in the initial dosage. Dependence is characterized by the repeated use of a substance for physical needs leading to increased tolerance and, when discontinued, results in physical withdrawal symptoms. Addiction is psychological dependence leading to persistent and increased use of the substance or behavior and, when discontinued, causes distress and urges to resume use despite adverse consequences. Withdrawal is a substance-specific cluster of signs and symptoms that occur when reducing or stopping the consistent use of the substance.
7-05. A patient who has been drinking a six-pack of beer each night to relax after work has been having trouble staying asleep through the night. When she wakes up at 2 a.m. she takes diphenhydramine 25 mg to help her fall back asleep, but this makes her sleep through the alarm clock and late for work. She also reports having a headache through midmorning until she gets her second cup of coffee. She thinks this is related to her alcohol use and has unsuccessfully tried to cut down several times in the past year. Which of the following disorders best describes this patient’s condition?
A. Alcohol use disorder
B. Withdrawal syndrome
C. Caffeine use disorder
D. Insomnia
Answer A. Alcohol use disorder is characterized by a pattern of use with significant impairment with two or more Diagnostic and Statistical Manual of Mental Disorders, Fifth Edition (DSM-5) criteria over the previous 12-month period. Withdrawal syndrome is a cluster of symptoms related to significantly reducing or stopping a substance or behavior. Caffeine intoxication requires a dose more than 250 mg in addition to a cluster of central nervous system excitatory symptoms shortly after the consumption. Insomnia is characterized by a persistent pattern of interrupted sleep and may be a primary problem or secondary concern induced by alcohol use.
7-06. The physiological process in which structural changes occur in the brain, increasing the likelihood of addiction over time and increasing the risk of relapse after a period of sobriety, is called:
A. Cognitive restructuring
B. Neuroadaptation
C. Neurogenesis
D. Cerebral ischemia
Answer B. Neuroadaptation is the physiologic process caused by a physical change in the body due to repeated exposure to a substance. Cognitive restructuring is a psychotherapeutic process of learning to identify and dispute irrational thoughts. Neurogenesis is the growth and development of neurons. Cerebral ischemia is brain tissue death caused by insufficient blood flow.
7-07. The PMHNP is evaluating a 45-year-old man in the emergency department; he has no past psychiatric history and presents with passive suicidal ideations and emotional lability. Which of the following tests is essential in helping formulate a preliminary diagnosis and treatment plan?
A. 12-lead EKG
B. Urine toxicology
C. Complete blood count
D. Liver profile
Answer B. Urine toxicology is essential because various drugs of abuse can induce various psychopathology, and all psychiatric diagnoses are made by exclusion. A 12-lead EKG will not provide insight into the presenting signs and symptoms but may guide medication selection. A complete blood count will not help explain the presenting symptoms, and any findings will require further investigation. The liver profile results will not explain the current symptom presentation but may raise the index of suspicion for further investigation.
7-08. A 43-year-old man is requesting medication-assisted treatment for addiction with Suboxone and wants to begin immediately to “get his life back.” What should the PMHNP do first?
A. Assess his current level and history of use
B. Prescribe Suboxone 2 mg SL daily to start now
C. Provide psychoeducation regarding Suboxone maintenance
D. Refer the patient to an inpatient detox facility
Answer A. Evaluating for substance use disorder requires an evaluation of the current level and history of use. Suboxone is not prescribed until the patient is in opioid withdrawal, otherwise a precipitated withdrawal may occur. Psychoeducation should be based on the findings from the evaluation, which must occur first. Before referring a patient for any therapy, it is important to know the extent of the problem.
7-09. Which of the following medications is preferred for a 21-year-old male with a history of polysubstance use disorder and difficulty with medication adherence?
A. Naltrexone 50 mg PO only when planning to use substances
B. Naltrexone 50 mg PO daily
C. Naltrexone 380 mg IM monthly
D. Methadone maintenance
Answer C. Naltrexone 380 mg IM offers the best option for medication nonadherence and harm reduction. Oral agents used at the patient’s discretion given his history of nonadherence are less likely to be effective. Beginning someone on methadone maintenance requires a high degree of adherence.
7-10. A patient who has been sober for 20 years is anticipating some major life changes in the next 3 months and is very worried about relapse. Which of the following actions should the PMHNP do first?
A. Refer the patient to Alcoholics Anonymous
B. Asses his coping strategies
C. Offer the patient naltrexone to be used in the Sinclair method
D. Refer the patient to inpatient rehab
Answer B. Assess coping strategies before prescribing treatment. The patient should have a thorough assessment that considers the risk factors for relapse before optimal relapse prevention strategies can be prescribed.
7-11. A 34-year-old man is inattentive, dysphoric, disinhibited, with impaired memory and psychomotor retardation. All of the following drugs of abuse may cause these symptoms except:
A. Alcohol
B. Opioids
C. Cannabis
D. Stimulants
Answer D. Stimulants of the central nervous system do not cause psychomotor retardation and memory impairment. Alcohol, opioids, and cannabis have central nervous system depressant properties and can cause inattention, dysphoria, disinhibition, cognitive impairment, and psychomotor retardation.
7-12. Which of the following agents could cause a patient to behave restlessly, agitated, grandiose, aggressive, and emotionally labile?
A. Amphetamines
B. Opioids
C. Marijuana
D. Ketamine
Answer A. Amphetamines stimulate the central nervous system and can cause restlessness, agitation, grandiosity, aggressive behavior, and emotional lability. Opioids, cannabinoids, and ketamine induce euphoric feelings and blunting of affect.
7-13. A 43-year-old man was involved in a motor vehicle collision in which he was found to have a blood alcohol concentration of 0.200. He was observed wandering around the crash site, slurring his words, and unable to provide a clear history. Emergency Medical Services brought him to the emergency department. The nurse triaged him to psychiatry for acute alcohol intoxication. What is the first priority for the PMHNP?
A. C-spine stabilization and head imaging
B. IV hydration
C. Explore the client’s willingness to enter rehab
D. Ask the patient when he had his last drink
Answer A. C-spine stabilization and head imaging are of paramount importance in this intoxicated man who was involved in a motor vehicle collision. Intoxication reduces his reliability as a historian and can impair his pain sensation. Physiological needs and safety take priority over other psychosocial needs.
7-14. In assessing a patient’s withdrawal symptoms from opioids, which evidence-based tool would be most appropriate?
A. Clinical Institute Withdrawal Assessment of Alcohol Scale Revised (CIWA-Ar)
B. Minnesota Multiphasic Personality Inventory (MMPI)
C. Clinical Opioid Withdrawal Scale (COWS)
D. Hamilton Rating Scale for Depression (HAM-D)
Answer C. The Clinical Opioid Withdrawal Scale (COWS) helps quantify the symptoms of opioid withdrawal to guide symptom management or Suboxone induction. The Clinical Institute Withdrawal Assessment of Alcohol Scale-Revised (CIWA-Ar) is used for alcohol withdrawal symptom assessment. The Minnesota Multiphasic Personality Inventory (MMPI) is a personality assessment. The Hamilton Rating Scale for Depression (HAM-D) screens for depression.
7-15. Which of the following immediate determinants related to relapse prevention is the biggest predictor of relapse?
A. Outcome expectancy
B. Negative emotional state
C. Ineffective coping skills
D. Previous history of lapse
Answer B. A negative emotional state is the most significant predictor of relapse. Outcome expectancy, ineffective coping skills, and previous history of relapse can also predict relapse but to a lesser degree than a negative emotional state.
7-16. The clinical management of patients differs depending on the substance-related syndrome exhibited. A patient with a history of alcohol use disorder who presents for a monthly injection of naltrexone is disinhibited, emotionally labile, inattentive, and impulsive and is likely in which of the following phases?
A. Acute withdrawal
B. Acute intoxication
C. Sobriety
D. Manic
Answer B. Acute intoxication is characterized by disinhibition, emotional lability, inattention, and impulsivity. Withdrawal symptoms include tremor, anxiety, hyperreflexia, diaphoresis, and autonomic hyperactivity. Sobriety is the quality of being congruent in mood and affect with good impulse control, and situationally appropriate responses. Manic is a state of abnormally elevated arousal, affect, and energy often with accompanying mood lability.
7-17. A patient presents for a follow-up visit and has a dual diagnosis of substance use disorder and major depressive disorder. He reports that he has been feeling increasingly anxious, having difficulty sleeping. He continues to attend his Alcoholics Anonymous meetings regularly, but reports he had two beers 3 days ago and feels very guilty. Which of the following best describes this patient’s situation?
A. Relapse
B. Withdrawal
C. Intoxication
D. Lapse
Answer D. A lapse is a brief pause in sobriety, which may or may not lead to relapse. Relapse is a reactivation of the disease or return to a behavior consistent with substance use disorder after a prolonged period of sobriety. Withdrawal is a substance-specific cluster of signs and symptoms that occur when reducing or stopping the consistent use of the substance. Intoxication is a state of near poisoning, manifesting signs of inebriation (disinhibition, impaired cognition, delayed neurological response time).
7-18. Which of the following substances would be most likely to cause withdrawal symptoms?
A. Alprazolam
B. Diazepam
C. Clonazepam
D. Chlordiazepoxide
Answer A. Alprazolam has an 11-hour half-life; medications with shorter half-lives are more likely to cause withdrawal symptoms if dependence develops. Diazepam, clonazepam, and chlordiazepoxide have half-lives >20 hours and are less likely to manifest withdrawal symptoms.
7-19. A patient with familial fatty liver disease and elevated liver enzymes secondary to alcohol use disorder is admitted for detox. Which of the following benzodiazepines should not be used?
A. Temazepam
B. Lorazepam
C. Oxazepam
D. Chlordiazepoxide
Answer D. Chlordiazepoxide is metabolized in the liver by cytochrome P-mediated oxidation. Temazepam, lorazepam, and oxazepam are metabolized by conjugation. They are less dependent on global liver function and are preferred in patients with liver disease.
7-20. A patient who has been sober for 7 days struggles with cravings and is fearful of relapse. He reports taking naltrexone daily. What agent should the PMHNP prescribe to help reduce his cravings?
A. Citalopram
B. Disulfiram
C. Buprenorphine
D. Thiamine
Answer A. Citalopram and other SSRIs have been found to decrease desirability, liking, and consumption of alcohol in alcohol-dependent drinkers. Given the patient presentation, prescribing an SSRI will help the patient without doing harm. Disulfiram is an aversive agent. Buprenorphine has been found to reduce cravings and withdrawal symptoms in patients with opioid dependence. Thiamine is indicated to prevent and mitigate Wernicke–Korsakoff syndrome associated with vitamin B1 deficiency, which is commonly found in patients with alcohol dependence.
8-01. A 66-year-old woman employed as a federal appeals lawyer is being seen by the PMHNP accompanied by her son, who has been concerned about progressive memory problems. Which of the following would be most helpful in distinguishing between common forgetfulness and cognitive impairment?
A. Brain MRI with and without contrast
B. Montreal Cognitive Assessment (MoCA)
C. Family history
D. Hamilton Depression Scale (HAM-D)
Answer B. The Montreal Cognitive Assessment is a psychometrically validated screening tool for mild cognitive impairment including visuospatial abilities, short-term recall, trail making, attention, concentration, language, and orientation. A brain MRI is helpful in identifying structural changes that may account for behavior if the affected brain region correlates with the impairment. Family history is important for stratifying risk factors and genetic predisposition but does not confirm a current diagnosis. The Hamilton Depression Scale is used to screen for major depressive disorder which can affect cognition, but the chief complaint was related to cognition.
8-02. A 62-year-old woman formerly employed as a receptionist reports progressive word-finding difficulty over the past 6 months that caused her so much anxiety she quit her job. Her daughter has become increasingly impatient waiting for her to finish her sentences but she has now become depressed due to the relational strain. She denies any other behavioral problems. Which of the following diagnostic tests would be most appropriate?
A. EEG
B. Brain MRI with and without contrast
C. Bilateral carotid Doppler study
D. Transthoracic echocardiogram
Answer B. A brain MRI is helpful in identifying structural changes which may account for behavior if the affected brain region correlates with the impairment such as chronic 137cerebrovascular accident affecting Wernicke’s or Broca’s area. An EEG identifies seizure-like activity while it is occurring. A bilateral carotid Doppler study may help stratify the risk for developing a cerebral vascular accident. A transthoracic echo may help stratify the risk for developing a cerebral vascular accident.
8-03. A 70-year-old retired male with a history of hypertension controlled with amlodipine, metoprolol, valsartan, and furosemide has been referred from his primary care provider for depression. Formerly an avid reader, the patient has lost interest in reading and has become increasingly isolative as he has been spilling things during his lunch club meetings and finds this embarrassing, which makes him feel self-conscious and anxious. In which portion of the mental status exam would the PMHNP expect to find a deficit?
A. Orientation to person, time, and place
B. Delayed recall
C. Abstract thinking
D. Intersecting pentagon
Answer D. Intersecting pentagons are used to assess visuospatial impairment, which reflects the function of the right parietal lobe, basal ganglia, and prefrontal cortex. Orientation to person, place, and time reflects the temporal lobe, frontal lobe, and cingulate cortex and is impaired in late phases of dementia. Delayed recall reflects the function of the hippocampus and the medial temporal lobe and is more common in Alzheimer’s dementia than frontotemporal dementia. Abstract thinking reflects the function of the frontal and prefrontal regions of the brain.
8-04. A 53-year-old man presents to the emergency department with complaint of sore throat progressively worsening for 3 days with intermittent fever. In triage he tells the RN that he has taken ibuprofen 400 mg 4 hours ago for the pain, and currently his vital signs are blood pressure 120/80, heart rate 88, respiratory rate 12, SPO2 96% on room air. While waiting for the results of the rapid strep test the patient becomes agitated, restless, confused, and increasingly difficult to redirect. He is transferred to the psychiatric emergency department because it is a locked unit. Which of the following is the most likely diagnosis?
A. Presenile dementia
B. Personality disorder
C. Mixed delirium
D. Mood disorder
Answer C. Mixed delirium is characterized by a cyclical manifestation of psychomotor retardation and agitation with a disturbance in consciousness; risk factors include infectious process, fever, and relocation. Dementia is a slowly progressive process that impairs cognition with the preservation of level of consciousness. Personality disorder is characterized by a historical pattern of rigid maladaptive coping mechanisms that cause distress. Mood disorder is a diagnosis of exclusion in which there are an insufficient number of symptoms to meet full criteria for major depressive disorder, bipolar disorder, or generalized anxiety disorder.
8-05. The PMHNP is consulted to see a 70-year-old female who was admitted to the skilled nursing facility 3 days ago from the hospital, where she was treated for urinary tract infection, pneumonia, and dehydration. The staff reports that the patient is restless and agitated, awake most of the night, intermittently sleeping during the day, and has poor PO intake. What should the PPMHNP do first?
A. Prescribe quetiapine 12.5 mg PO at bedtime and every 6 hours as needed for agitation
B. Physical exam and diagnostic tests (complete blood count, basic metabolic profile, and urinalysis)
C. Transfer the patient to the ED
D. Call family members for collateral information on her baseline functioning
Answer B. Physical exam and diagnostic tests are used to rule out delirium given the patient’s history, age, and clinical presentation. Prescribing medication can worsen delirium and mask symptoms of an underlying cause. Transferring the patient to the ED may worsen or trigger delirium. Obtaining collateral information is helpful in helping to establish a baseline pattern of function, but the acute problem of delirium must be addressed and ruled out first.
8-06. A 55-year-old woman reports to the PMHNP that since she accepted early retirement last year she finds that her memory has gotten worse and she has trouble sleeping at night. She denies any medical problems. What should the PMHNP do first?
A. Perform a Mini-Mental State Exam and Confusion Assessment Methods Instrument
B. Prescribe mirtazapine 15 mg PO at bedtime to help her sleep
C. Encourage her to volunteer with something she feels passionate about
D. Refer her to her primary care provider for a physical exam
Answer A. A formalized assessment such as the Mini-Mental State Exam and Confusion Assessment Methods Instrument provides a reproducible means of establishing baseline function and distinguishing delirium from reversible cognitive impairment. Mirtazapine is indicated for major depressive disorder and anxiety but both are a diagnosis of exclusion and are lower priority than delirium. Encouraging volunteer activity may be a good way to develop psychological activation in a patient with a mood disorder, but there is no evidence to support this diagnosis. Referring the patient to her primary care provider for a physical exam does negates the PMHNP’s responsibility to evaluate for neurocognitive disorders.
8-07. Karen is a 68-year-old former legal secretary who enjoys an active social life with her husband. She feels she has something wrong with her medically and has seen many specialists over the year who have found nothing wrong with her. Her husband reports that she continually repeats the same five stories over and over again as if she is telling them for the first time. On two occasions in the last year she has gotten lost coming home from the supermarket and was found by the police a half-mile from her former place of employment. What is the most likely diagnosis?
A. Vascular dementia
B. Huntington’s disease
C. Creutzfeldt–Jakob disease
D. Mixed delirium
Answer A. Vascular dementia is the second most common dementia, and is caused by progressive cardio-/cerebrovascular disease and manifests in cognitive decline and plateau phases in which previously lost function is not regained. Huntington’s dementia is a subcortical disease characterized by motor abnormalities including psychomotor slowing, choreoathetoid movements, and executive dysfunction complicated by impaired language, memory, and insight later in the disease process. Pick’s disease is also known as frontotemporal dementia, and is characterized by a change in personality and cognitive decline; a subtype is Kulver-Bucey syndrome, which is manifested by uninhibited cheerfulness, hypersexuality, and hyperorality. Creutzfeldt–Jakob disease is a precipitous onset cognitive decline that rapidly progress to death; symptoms include fatigue and cognitive impairment and eventually aphasia, apraxia, emotional lability, and psychosis. Mixed delirium is characterized by a cyclical manifestation of psychomotor retardation and agitation with a disturbance in consciousness; risk factors include infectious process, fever, and relocation.
8-08. Which type of dementia is characterized by a gradual onset and progressive decline without focal neurological deficits?
A. Alzheimer’s disease
B. Pick’s disease
C. Vascular dementia
D. Huntington’s disease
Answer A. Alzheimer’s disease is the most prevalent type of dementia, and is characterized by a gradual onset and progressive decline without focal neurological deficits. Pick’s disease is also known as frontotemporal dementia, and is characterized by a change in personality and cognitive decline; a subtype is Kulver-Bucey syndrome, which is manifested by uninhibited cheerfulness, hypersexuality, and hyperorality. Vascular dementia is the second most common dementia, and is caused by progressive cardio-/cerebrovascular disease and manifests in cognitive decline and plateau phases in which previously lost function is not regained. Huntington’s dementia is a subcortical disease characterized by motor 138abnormalities including psychomotor slowing, choreoathetoid movements, and executive dysfunction complicated by impaired language, memory, and insight later in the disease process.
8-09. A 72-year-old man with a history of hypertension, and type 2 diabetes mellitus has become increasingly disinhibited and flirts with his home health aide. He demands she kiss him or he will pee on himself. She is sometimes able to distract him with cookies but his children often object because it will worsen his blood sugar control. Which of the following is the most likely diagnosis?
A. Alzheimer’s disease
B. Frontotemporal dementia
C. Vascular dementia
D. Creutzfeldt–Jakob disease
Answer B. Pick’s disease is also known as frontotemporal dementia, and is characterized by a change in personality and cognitive decline; a subtype is Kulver-Bucey syndrome, which is manifested by uninhibited cheerfulness, hypersexuality, and hyperorality. Alzheimer’s disease is the most prevalent type of dementia, and is characterized by a gradual onset and progressive decline without focal neurological deficits. Vascular dementia is the second most common dementia, and is caused by progressive cardio-/cerebrovascular disease and manifests in cognitive decline and plateau phases in which previously lost function is not regained. Creutzfeldt–Jakob disease is a precipitous onset cognitive decline that rapidly progress to death; symptoms include fatigue and cognitive impairment and eventually aphasia, apraxia, emotional lability, and psychosis.
8-10. A 50-year-old man with a diagnosis of Parkinson’s disease is admitted to the hospital after tripping and falling and breaking his hip while on a business trip, where he was inspecting a slaughterhouse. The third day after surgery the patient is having tremors, intermittent slurred speech, dysarthria, and dysgraphia. The orthopedist is attributing his symptoms to slow postoperative progression complicated by having not taken his antiparkinsonian medications for 2 days and the effects of narcotics. Psychiatry was asked to see the patient for mood disorder and possible delirium to aid in planning discharge to subacute rehab. On exam the patient is unable to name the day, month, or year; has poor three-object registration and recall, scoring 12 of 30 on a mental state exam; his speech fluency and attention span are incongruent with his baseline; and he has no decline in his level of consciousness. Which of the following is among the primary differential diagnoses?
A. Major depressive disorder
B. Presenile dementia
C. Delirium
D. Creutzfeldt–Jakob disease
Answer D. Creutzfeldt–Jakob disease is a precipitous onset cognitive decline that rapidly progress to death; symptoms include fatigue and cognitive impairment and eventually aphasia, apraxia, emotional lability, and psychosis. Major depressive disorder is a syndrome identified by five of nine symptoms lasting for 2 weeks, with more than one of the nine symptoms being either depressed mood or anhedonia. Other symptoms can include a change in appetite with weight gain or weight loss, impaired cognition, fatigue, feelings of low self-worth, inattention, and recurrent thoughts of death. Dementia is a constellation of signs and symptoms characterized by a gradual onset of multiple impairments in executive function, intellect, problem-solving, and alteration in memory with the preservation of level of consciousness. Delirium is a constellation of symptoms marked by an acute onset (hours to days) causing short-term decline in cognition with a disturbance in consciousness and inattention.
8-11. A 68-year-old man with no past psychiatric history but with a history of poly substance use disorder is admitted for psychosis (disorganized thoughts, rapid pressured speech, persistent rumination, paranoid delusions, and apathy). Which of the following tests would be most important in determining the etiology of his new onset symptoms?
A. Urine toxicology screening
B. HIV testing
C. Mental state exam
D. Urine culture
Answer B. HIV-associated dementia, a neurocognitive disorder with parenchymal abnormalities visualized on MRI, manifests a progressive decline with associated symptoms including obsessive-compulsive disorder, generalized anxiety disorder, depression, and mania. Urine toxicology, while essential in the initial evaluation of a person with altered mental status, is of low yield in this case as polysubstance use disorder is known. A mental state exam has at least partially been completed given the clinical findings identified in the question. Urine culture is important in the evaluation of delirium but given the high-risk lifestyle factors HIV must be ruled out as the treatment requires antiretroviral therapy.
8-12. When evaluating a 78-year-old man, the PMHNP notices the patient frequently leans in with the right side of his head and frequently asks for things to be repeated. His daughter is very worried that he has dementia or depression because he has also become increasingly isolated, as well as increasingly irritable as he believes people are yelling at him out of frustration of not being heard. The daughter is tearful, anxious, and frustrated. What should the PMHNP do first?
A. Separate the patient from the daughter and interview him alone
B. Examine the patient’s ear canals
C. Order a CT scan of the head
D. Provide anticipatory guidance regarding aging parents to the daughter
Answer B. The patient is leaning in to hear and asking for things to be repeated but there are no signs and symptoms to indicate dementia or depression. Separating the patient from his daughter may be appropriate but physical needs need to be evaluated first. Head CT scan is not indicated based on the presenting symptoms and increases the risk of incidental findings. Anticipatory guidance should be based on actual problems to help provide for future care needs.
8-13. When providing anticipatory guidance to the loved ones of a 68-year-old woman who has been diagnosed with dementia, what is the most important information to convey?
A. The importance of taking medications exactly as prescribed
B. Safety planning regarding wandering, driving, and fire and fall risk
C. Implementing advance directives for end of life care
D. Encouraging the patient’s autonomy in expressing their wishes
Answer B. Safety planning regarding wandering, driving, and fire and fall risk is warranted. Medication adherence is important but a lesser priority in this case. Advance directives for end of life care is an important discussion but less priority than attending to immediate safety needs. Helping the patient retain autonomy is an important ethical principle to govern care, but the current healthcare system puts priority on safety planning.
8-14. Which of the following conditions frequently occurs in patients with dementia but may improve as the condition declines?
A. Psychosis
B. Personality disorders
C. Depression
D. Delirium
Answer C. Depression commonly occurs in patients with dementia but the symptoms are reduced as the patient loses self-awareness. Psychosis is more common in patients with HIV-associated dementia and can worsen as dementia progresses. Personality disorder is characterized by a historical pattern of rigid maladaptive coping mechanisms that cause distress. Delirium is characterized by a cyclical manifestation of psychomotor retardation and agitation with a disturbance in consciousness; risk factors include infectious process, fever, and relocation. Psychosis is characterized by the onset of delusions, hallucinations, or disorganized thinking and speech, or grossly disorganized or catatonic behavior.
8-15. Jose is a 48-year-old veteran of Operation Iraqi Freedom, during which he sustained a traumatic brain injury that left him unconscious for 7 days. He has progressively regained function and is able to perform his instrumental activities of daily living most days. His wife is worried because he has periods where she does not recognize him as he becomes irritable, fails to remember important dates and events, and gets headaches so bad that he says he wishes he had died in the war. What is the priority action for the PMHNP to take during this office visit?
A. Provide anticipatory guidance and coping mechanisms to the patient and his wife
B. Refer the patient to the ED for suicidal ideation
C. Prescribe antipsychotic medications to be used when the patient becomes irritable
D. Refer the patient to a neurologist
Answer A. 139Providing anticipatory guidance and coping mechanisms to the patient and his wife regarding traumatic brain injuries can help mitigate the impact of some of the symptoms. Referring the patient to the emergency department for suicidal ideations is inappropriate as the patient is not acutely suicidal. Prescribing antipsychotic medication to be used when the patient becomes irritable may be appropriate but nonpharmacological measures as first line are preferred. Referring the patient to a neurologist is not warranted as the primary problem is the sequelae of a traumatic brain injury and family system involvement for ongoing care.
8-16. The husband of a 68-year-old woman with major depressive episodes is concerned about cognitive decline in his wife. Which of the following statements best addresses his concern regarding this symptom?
A. The cognitive decline is reversible once the depression is treated
B. It will be important to plan for her long-term care needs
C. I can prescribe a medication to enhance her memory
D. It is normal for you to be worried about your wife
Answer A. The cognitive decline associated with depression is reversible once the depression is treated. Long-term care planning for cognitive impairment secondary to a mood disorder is not eminently indicated. Prescribing medication to enhance memory is not indicated in a patient with a primary mood disorder that is insufficiently treated. Placating the family member without sufficient psychoeducation is not the best answer.
8-17. The primary distinguishing feature between dementia and delirium is:
A. Onset of symptoms and duration
B. Disturbance in consciousness
C. Age of the patient
D. Comorbid conditions
Answer B. A disturbance in consciousness is the primary distinguishing feature between dementia and delirium. All delirium is traditionally acute onset and dementia is chronic onset; certain dementias can be more precipitous. Patients of any age can have either delirium or dementia. Patients with both dementia and delirium can have comorbid conditions.
8-18. The primary treatment for delirium is:
A. Antipsychotic medications
B. Mood stabilizing medications
C. Identifying and treating the underlying cause
D. Anticipatory guidance and psychoeducation
Answer C. Identifying and treating the underlying cause is the primary treatment for dementia. Adding medications to a patient with delirium with an unknown underlying cause may worsen or compound the delirium as polypharmacy is another causative factor. Anticipatory guidance and psychoeducation is appropriate when tailored to a specific condition.
8-19. Delirium characterized by periods of agitation and restlessness is known as:
A. Hypoactive delirium
B. Hyperactive delirium
C. Mixed delirium
D. Hypomania
Answer C. Mixed delirium is characterized by cycling through psychomotor agitation and retardation ranging from apathy to hypervigilance. Hypoactive delirium is characterized by psychomotor retardation and apathy. Hyperactive delirium is characterized by psychomotor agitation, restlessness, and hypervigilance. Mania is characterized by grandiosity, a reduced need for sleep, being more talkative than usual, flight of ideas and racing thoughts, distractibility, and psychomotor agitation with an increase in goal-directed, pleasure-seeking behaviors.
8-20. Delirium characterized by apathy, psychomotor retardation, and inattentiveness is known as:
A. Hypoactive delirium
B. Hyperactive delirium
C. Mixed delirium
D. Mania
Answer A. Hypoactive delirium is characterized by psychomotor retardation and apathy. Hyperactive delirium is characterized by psychomotor agitation, restlessness, and hypervigilance. Mixed delirium is characterized by cycling through psychomotor agitation and retardation ranging from apathy to hypervigilance. Mania is characterized by grandiosity, reduced need for sleep, more talkative than usual, flight of ideas and racing thoughts, distractibility, and psychomotor agitation with an increase in goal-directed, pleasure-seeking behaviors.
9-01. A 19-year-old male is attending his post hospital discharge appointment where he was diagnosed with psychosis. He has been taking risperidone 4 mg by mouth twice daily. Which of the following is the top priority during this first encounter with the PMHNP?
A. Baseline EKG, weight, hemoglobin A1C
B. Establish therapeutic alliance
C. Draw a prolactin level
D. Administer a long-acting injectable
Answer B. Establishing a therapeutic alliance is the top priority in order to proceed with any further care for this patient. Baseline EKG, weight, and hemoglobin A1C while essential may not be completed unless the patient therapeutic alliance is established. Prolactin level is expected to be elevated in a patient taking risperidone and other long-acting injectables and may provide insight into compliance, but other factors can cause this level to rise. Administering a long-acting injectable is preferred in patients who are medication nonadherent.
9-02. The PMHNP is evaluating a 30-year-old female who reports intermittent auditory hallucinations in which a familiar female voice is commenting on her as she eats. In an effort to deal with the voices she has been using headphones and listens to podcasts and music all the time, but it is interfering with her ability to do her work. Which of the following questions would help in formulating a diagnosis?
A. “How long have you been experiencing the voices?”
B. “Do the voices bother you?”
C. “Are you able to ignore the voices?”
D. “What has helped you deal with the voices?”
Answer A. Asking how long the patient has been experiencing the voices allows the PMHNP to determine a more accurate diagnosis and can inform long-term care options. Asking if the voices are bothersome may further inform the assessment. Asking if the patient is able to ignore the voices informs what specific coping strategies have been successful or unhelpful in the past. Inquiring of past helpful coping strategies is beneficial in empowering patients to problem solve.
9-03. A couple presents with their 18-year-old son who was referred by his primary care provider for evaluation of symptoms related to perceptual disturbances, oddly related interpersonal communication, neglect of basic hygiene, and increasing isolative behaviors. These symptoms are most consistent with:
A. Active psychosis
B. Schizophrenia prodrome
C. Residual psychosis
D. Schizophrenia
Answer B. Schizophrenia prodrome is characterized by odd beliefs, ideas of reference, unusual perceptual experience, negative symptoms, and deterioration of function prior to the onset of active psychosis. Active psychosis is characterized by delusions, hallucinations, disorganized speech, disorganized behavior, and/or pervasive negative symptoms. Residual phase psychosis is characterized by persistent functional impairment, abnormalities of affect, impaired cognition, and impaired communication.
9-04. A 28-year-old female is referred from her employee assistance program from a tech company for poor hygiene and erratic behaviors interfering with her work. The patient states her company is spying on her and setting her up to fail for the last 9 months. While speaking she stops herself and begins to talk back to someone who is not present. These symptoms are most consistent with:
A. Psychosis
B. Delusions
C. Hallucinations
D. Depression
Answer A. 150Psychosis is characterized by paranoid delusions, poor self-care, erratic behavior, and auditory hallucinations persisting for >6 months. Delusions are deeply held beliefs despite evidence to the contrary. Hallucinations are perceptual disturbances involving one of the five senses. Depression is a mood disorder that can cause psychotic symptoms including hallucinations, delusions, and cognitive impairment but there is no mention of a primary mood disorder in this question.
9-05. A 55-year-old female recently accepted a severance package for early retirement from her job as a fashion buyer for a major retail store. Over the past 6 months she has increasingly become isolated, neglecting her self-care, and collecting various things of no real value. Her appearance is unkempt, and her affect is flat. Before being called back to the consultation room the PMHNP notices that the patient seems to be talking to herself audibly. During the consultation the patient makes poor eye contact and wants frequent reassurance that no one is listening in. Which of the following would be the best action for the PMHNP to take?
A. Send the patient to the psychiatric emergency department
B. Speak in a calm reassuring voice and evaluate for comorbid conditions
C. Prescribe an antipsychotic medication for atypical psychosis
D. Order an MRI of the brain to rule out a mass
Answer B. Speaking in a calm, reassuring voice and evaluating for comorbid conditions can help the PMHNP build a therapeutic alliance and gather essential information to formulate an accurate diagnosis. Sending the patient to the emergency department may rupture the therapeutic alliance, and in this case, there is no indication of imminent threat to self or others. Prescribing an antipsychotic may be appropriate but the diagnosis of atypical psychosis is not definitive. Head imaging studies are often low yield and should be reserved for atypical presentations that require further assessment.
9-06. When providing anticipatory guidance for the parents of a 23-year-old male with schizophrenia, which of the following statements is most accurate?
A. Medications are most effective for the anhedonia, apathy, and lack of motivation
B. Medications are most effective for hallucinations, disorganization, and delusions
C. Schizophrenia is caused by the use of marijuana
D. Most patients with schizophrenia can live a normal life without medication
Answer B. Medications are most effective for positive symptoms. Medications are least effective for negative symptoms. Schizophrenia is not caused by marijuana, but marijuana can induce psychosis, paranoia, and disorganized behavior, which should resolve when the substance is cleared from the body. Most patients with schizophrenia are on lifelong medication and life is often difficult despite the best treatment.
9-07. A 61-year-old female is referred to the psychiatric emergency department with complaints of abdominal pain, nausea, and bloody stools. Despite an extensive evaluation, no underlying cause for her symptoms can be identified and her hemoglobin is normal. The patient insists there is something physically wrong with her. Which of the following best describes her condition?
A. Persecutory delusion
B. Grandiose delusion
C. Jealous delusion
D. Somatic delusion
Answer D. With somatic delusion, content is focused on bodily functions and perceived ailments despite evidence to the contrary or absence of evidence for the physical complaint. With persecutory delusions, content is focused on feeling targeted, victimized, and/or singled out by authority figures. Grandiose delusions are focused on the patient’s great talent, status, skills, or knowledge. Jealous delusions are focused on infidelity with attempts to control the behavior of others they believe are being unfaithful.
9-08. A 22-year-old man is referred to the psychiatric emergency department for suicidal ideation. Upon interview the patient states that he is not trying to kill himself, and he really can fly if only people would let him. The patient is emphatic that he is not manic because he is “the best sleeper in the world.” The patient has a bright reactive affect and is pressured at times. Which of the following best describes his condition?
A. Grandiose delusion
B. Hallucinations
C. Erotomaniac delusions
D. Persecutory delusions
Answer A. Grandiose delusions are focused on the patient’s great talent, status, skills, or knowledge. Hallucinations are a perceptual disturbance involving one of the five senses. Erotomania is a delusion that someone is in love with the patient or the patient feels infatuated with a person and trys to get them to reciprocate their passionate feelings, becoming enraged when they perceive rejection. During persecutory delusions, content is focused on feeling targeted, victimized, and/or singled out by authority figures.
9-09. A 32-year-old man presents to the emergency department with acute onset disorganization, paranoid delusions, and visual hallucinations of flying bugs, which he tries to crush because he thinks they contain listening devices. The patient is accompanied by the police, who were called because the man was smashing things in a store. Which of the following can the PMHNP diagnose during the initial encounter?
A. Schizophrenia
B. Schizophreniform disorder
C. Brief psychotic disorder
D. Schizoaffective disorder
Answer C. Brief psychotic disorder is characterized as disorganization accompanied by either delusions, hallucinations, or disorganized speech (marked with frequent derailment, incoherence, or world salad) lasting <30 days. Schizophrenia includes the symptoms of psychosis (disorganized speech, delusions, hallucinations) lasting >6 months. Schizophreniform disorder is characterized by psychotic symptoms lasting more than 30 days, but <6 months. Schizoaffective disorder includes psychotic symptoms with mutually exclusive manic symptoms.
9-10. A 33-year-old female with history of schizoaffective disorder was brought to the emergency department by police for disruptive behavior, auditory hallucinations, and self-neglect. The patient screams that she had stopped taking her medication. The PMHNP orders Haldol 5 mg PO, which the patient willingly accepts. An hour later the patient demonstrates an involuntary upward deviation of the eyes, and hiccoughs. Which of the following should the PMHNP do first?
A. Draw stat labs for complete blood count, creatinine phosphokinase
B. Stat EKG
C. Haldol 5 mg IM stat
D. Benadryl 25 mg IV push
Answer D. Benadryl 25 mg IV push is an antihistamine used for anticholinergic effect, indicated for use in acute extrapyramidal symptoms such as dystonia, oculogyric crisis, and diaphragm spasm. Stat labs are not the priority in this patient with an oculogyric crisis as it will not change the management or solve the problem. Stat EKG does not address the presenting problem. Haldol may worsen the problem and is likely the cause of the problem.
9-11. A 45-year-old man who takes olanzapine 5 mg twice daily for psychosis reports a perpetual sense of restlessness and an inability to sit still. He says, “sometimes it gets so bad I want to jump out of my own skin.” Which of the following is the most likely explanation?
A. Dystonia
B. Akinesia
C. Akathisia
D. Tardive dyskinesia
Answer C. Akathisia is the inability to remain still, includes motor restlessness, and is often mistaken for anxiety but described in similar terms. Dystonia is an involuntary muscle spasm due to dopamine blocking agents such as antipsychotic medication. Akinesia is the absence of movement or the difficulty in starting movements with associated flat affect and apathy, and is related to dopamine-blocking agents such as antipsychotic medications. Tardive dyskinesia is a constellation of involuntary movements in a rhythmic pattern that is potentially irreversible due to dopamine blocking agents.
9-12. The PMHNP is evaluating a 37-year-old man who takes a long-acting injectable paliperidone palmitate, who reports that he is glad to not have to take a pill every day. He reports that often, for the first few days after he gets his injection, he notices that he walks slower than usual and with a shuffling gait, feels stiffness in his joints, and people think he is depressed but the symptoms dissipate as the month progresses. Which of the following best describes this condition?
A. Dystonia
B. Akinesia
C. Akathisia
D. Pseudo-Parkinson’s
Answer D. Pseudo-Parkinson’s is a movement disorder exhibited by a shuffling gait, motor slowing (bradykinesia), masked faces, and low frequency tremor at rest. Akathisia is characterized by the inability to remain still and motor restlessness, and is often mistaken for anxiety but described in similar terms. Dystonia is an involuntary muscle spasm due to dopamine blocking agents such as antipsychotic medication. Akinesia is the absence of movement or the difficulty in starting movements with associated flat affect and apathy, and is related to dopamine-blocking agents such as antipsychotic medications.
9-13. A condition characterized by abnormal involuntary movements in a rhythmic pattern often affecting the mouth, tongue, and jaw that is potentially irreversible is known as?
A. Tardive dyskinesia
B. Neuroleptic malignant syndrome
C. Pseudo-Parkinson’s
D. Dystonia
Answer A. Tardive dyskinesia is a constellation of involuntary movements in a rhythmic pattern that is potentially irreversible due to dopamine-blocking agents. Neuroleptic malignant syndrome is a rare life-threatening reaction to neuroleptic antipsychotic medication, and is associated with high fever, delirium, muscle rigidity, autonomic instability, and diaphoresis. Pseudo-Parkinson’s is a movement disorder exhibited by a shuffling gait, motor slowing (bradykinesia), masked faces, and low frequency tremor at rest. Akathisia is charcterized by the inability to remain still and motor restlessness, and is often mistaken for anxiety but described in similar terms. Dystonia is an involuntary muscle spasm due to dopamine blocking agents such as antipsychotic medication.
9-14. An uninsured patient was who discharged from the hospital on haloperidol 10 mg twice daily presents to the PMHNP for a 2-week follow-up medication check visit. On exam the patient has tachypnea, tachycardia, and tremors, the skin is hot to the touch, and is overall very rigid. What should the PMHNP do first?
A. Prescribe Benadryl 25 mg PO stat
B. Call 911 for transfer to the emergency department
C. Discontinue the Haldol
D. Attempt to establish peripheral IV access
Answer B. Call 911; the patient is in acute physiological distress with a high index of suspicion for neuroleptic malignant syndrome and is in need of acute medical attention. Prescribing medication should not take priority over advanced life support. Stopping Haldol is correct but does not take priority over getting the patient to the hospital. Attempting to establish IV access can occur after 911 has been called.
9-15. When providing psychoeducation to the parents of a 20-year-old male with psychosis, which of the following is considered a good prognostic indicator?
A. Early onset
B. Substance induced
C. First degree family member with schizophrenia
D. Delayed treatment with antipsychotics
Answer B. Substance-induced psychosis offers the best prognosis because often, when the substance is stopped and cleared from the body, the patient can return to baseline. Early onset, first-degree relative, and delayed treatment of psychotic symptoms are all poor prognostic indicators.
9-16. A 36-year-old man has been readmitted five times in the past year for medication nonadherence despite the use of a long-acting injectable antipsychotic for schizophrenia. The patient is often brought in by police when bystanders report reckless behavior. Which of the following would provide him with the best chance of reduced morbidity and autonomous living?
A. State hospitalization
B. Supportive housing
C. Court ordered assertive community treatment
D. Psychosocial clubhouse
Answer C. Court-ordered assertive community treatment allows the patient to live in the community and have multidisciplinary care delivered if the patient fails to come to appointments including long-acting injectables. State hospitalization inhibits autonomy. Supportive housing is insufficient for medication-nonadherent patients. A psychosocial clubhouse relies on the patient’s own initiative to present and does not administer medications.
9-17. A 30-year-old woman is convinced her neighbor is hypnotizing her into performing sexual acts in her sleep because he is turned on by her doing yoga for the last 3 months. She presented to the emergency department requesting a sexual assault forensic exam to prove her claim. The examiner finds her story bizarre and requests a psychiatric evaluation for capacity to consent to forensic examination. When obtaining collateral information, the police reveal that the patient has a restraining order against her from her neighbor for harassment. Which of the following best describes the patient’s condition?
A. Psychosis
B. Erotomanic delusion
C. Somatic delusion
D. Posttraumatic stress disorder
Answer B. Erotomanic delusion content is focused on the belief that someone is in love with the patient, the love is idealized and may contain sexual content without evidence to support this, and the objectified lover may not even be aware of the patient. Psychosis is characterized by paranoid delusions, poor self-care, erratic behavior, hallucinations, and disorganization. Somatic delusion content is focused on perceived debilitated body functions or ailments despite any evidence to support the belief. Posttraumatic stress disorder is characterized by flashbacks, nightmares, severe anxiety, and persistent ruminations of a traumatic event.